Docsity
Docsity

Prepare for your exams
Prepare for your exams

Study with the several resources on Docsity


Earn points to download
Earn points to download

Earn points by helping other students or get them with a premium plan


Guidelines and tips
Guidelines and tips

Emergency Medical Response: Patient Care and Assessment, Exams of Laboratory Practices and Management

Answers to various patient care scenarios, covering topics such as seizures, hypoglycemia, abdominal pain, anaphylactic shock, triage, and use of aeds. It also emphasizes the importance of scene size-up, patient care priorities, and infection control.

Typology: Exams

2023/2024

Available from 04/04/2024

A-Grade
A-Grade 🇺🇸

701 documents

1 / 182

Toggle sidebar

Related documents


Partial preview of the text

Download Emergency Medical Response: Patient Care and Assessment and more Exams Laboratory Practices and Management in PDF only on Docsity! NREMT Practice Test Bank Questions with Verified Solutions for 2024 Update. Hypoglycemia and acute ischemic stroke can present similarly because: • A: both oxygen and glucose are needed for brain function. • B: the majority of stroke patients have a history of diabetes. • C: the most common cause of a stroke is hypoglycemia. • D: they are both caused by low levels of glucose in the blood. - ANS • a: both oxygen and glucose are needed for brain function. Reason: Although stroke and hypoglycemia are two distinctly different conditions, their signs and symptoms are often similar. This is because the brain requires both oxygen and glucose to function normally. An acute ischemic stroke is caused by a lack of oxygen to a part of the brain due to a blocked cerebral artery, whereas hypoglycemia (low blood glucose level) deprives the entire brain of glucose. In either case, the patient presents with signs of impaired brain function (i.e., slurred speech, weakness, altered mental status). Both conditions may lead to permanent brain damage or death if not treated promptly. When dealing with an emotionally disturbed patient, you should be MOST concerned with: • A: gathering all of the patient's medications. • B: safely transporting to the hospital. • C: whether the patient could harm you. • D: obtaining a complete medical history. - ANS You selected C; this is correct! Reason: When managing any patient with an emotional or psychiatric crisis, your primary concern is your own safety. Safely transporting the patient to the hospital is your ultimate goal. If possible, you should attempt to obtain a medical history and should take any of the patient's prescribed medications to the hospital. However, this should not supersede your own safety or interfere with safely transporting the patient. You are at the scene where a man panicked while swimming in a small lake. Your initial attempt to rescue him should include: • A: rowing a small raft to the victim. • B: reaching for the victim with a long stick. • C: throwing a rope to the victim. • D: swimming to the victim to rescue him. - ANS You selected B; this is correct! Reason: General rules to follow when attempting to rescue a patient from the water include "reach, throw, row, and then go." In this case, you should attempt to reach the victim by having him grab hold of a large stick or similar object. If this is unsuccessful, throw the victim a rope or flotation device (if available). If these are not available, row to the patient in a small raft (if available). Going into the water to retrieve the victim is a last resort. The rescuer must be a strong swimmer because patients who are in danger of drowning are in a state of blind panic and will make every attempt to keep themselves afloat, even if it means forcing the rescuer underwater. How should you classify a patient's nature of illness if he or she has a low blood glucose level, bizarre behavior, and shallow breathing? • A: Behavioral emergency • B: Altered mental status • C: Respiratory emergency • D: Cardiac compromise - ANS The correct answer is B; Reason: The nature of illness (NOI) is the medical equivalent to mechanism of injury (MOI). Altered mental status should be the suspected NOI in any patient with any fluctuation in level of consciousness, which can range from bizarre behavior to complete unresponsiveness. Causes of an altered mental status include hypo- or hyperglycemia, head trauma, stroke, behavioral crises, drug overdose, and shock, among others. A young female is unresponsive after overdosing on an unknown type of drug. Her respirations are slow and shallow and her pulse is slow and weak. Which of the following drugs is the LEAST likely cause of her condition? • A: Seconal • B: Heroin • C: Cocaine • D: Valium - ANS The correct answer is C; Reason: Of the drugs listed, cocaine would be the least likely cause of the patient's condition. Cocaine is a central nervous system (CNS) stimulant; you would expect her to be hypertensive, tachycardia, tachypnea, and perhaps even violent. Heroin, Valium, and Secondly are all CNS depressants and could explain her condition. Heroin is an illegal narcotic (opiate), Valium is a benzodiazepine sedative-hypnotic drug, and secondly is a barbiturate. Narcotics, benzodiazepines, and barbiturates are all CNS depressants. When taken in excess, they cause a decreased level of consciousness, respiratory depression, bradycardia, and hypotension. Activated charcoal is contraindicated for a patient who is: • A: conscious and alert and has ingested a large amount of Motrin. • B: emotionally upset and has ingested two bottles of aspirin. • C: agitated and claims to have ingested a bottle of Tylenol. • D: awake and alert and has swallowed a commercial drain cleaner. - ANS You selected D; this is correct! Reason: Activated charcoal adsorbs (sticks to) many ingested substances, preventing them from being absorbed into the body by the stomach or intestines. In some cases, you may give activated charcoal to patients who have ingested certain substances, if approved by medical control or local protocol. Activated charcoal is contraindicated for patients who have ingested an acid or alkali (i.e., drain cleaner) or a petroleum product • C: meningitis. • D: tuberculosis. (- ANS You selected C; this is correct! Reason: Meningitis is an inflammation of the protective coverings of the brain and spinal cord (meninges). Common signs and symptoms of meningitis include fever, headache, neck stiffness (nuchal rigidity), and vomiting. An altered mental status is common in severe cases. Meningococcal meningitis, caused by a bacterium, is the most contagious and potentially fatal type of meningitis. The patient's signs and symptoms are not consistent with acute stroke, tuberculosis (TB), or influenza (the flu). Although fever is common with both TB and the flu, neither causes neck stiffness. Acute stroke may be associated with a headache, especially a hemorrhagic stroke; however, stroke patients typically do not have a fever. All of the following are signs of gastrointestinal bleeding, EXCEPT: • A: melena. • B: hematemesis. • C: tachycardia. • D: hemoptysis. - ANS You selected D; this is correct! Reason: Signs and symptoms of gastrointestinal (GI) bleeding include abdominal pain; vomiting blood (hematemesis); the passage of dark, tarry stools (melena); and bright red rectal bleeding (hematochezia). If blood loss is significant, the patient may have signs of shock (egg, tachycardia, diaphoresis, tachypnea, and hypotension). Hemoptysis (coughing up blood) is a sign of a pulmonary injury, not GI bleeding. A 24-year-old female presents with a rash to her left leg and swollen, painful knee joints. She tells you that she and her friends returned from a hiking trip in the mountains a week ago. She is conscious and alert with a blood pressure of 112/62 mm Hg, a pulse of 84 beats/min, and respirations of 14 breaths/min. Her symptoms are MOST likely the result of: • A: Rocky Mountain spotted fever. • B: tetanus. • C: Lyme disease. • D: a localized allergic reaction. - ANS You selected C; this is correct! Reason: The patient's symptoms and her history of a recent hiking trip are consistent with Lyme disease, which was the result of a tick bite. Ticks can carry two infectious diseases: Lyme disease and Rocky Mountain spotted fever. Both are spread through the tick's saliva, which is injected into the skin when the tick attaches itself. The first symptom of Lyme disease, a rash that may spread to several parts of the body, begins about 3 days after the bite of an infected tick. The rash may eventually resemble a target bull's-eye pattern in one third of patients. After a few more days or weeks, painful swelling of the joints, particularly the knees, occurs. If recognized and treated promptly with antibiotics, many patients recover completely. Rocky Mountain spotted fever, which is not limited to the Rocky Mountains, occurs within 7 to 10 days after being bitten by an infected tick. Its symptoms include nausea, vomiting, headache, weakness, paralysis, and possibly cardiopulmonary failure. A 50-year-old woman who is conscious and alert complains of a severe migraine headache. When caring for her, you should generally avoid: • A: transporting her in a supine position. • B: shining a light into her pupils. • C: dimming the lights in the ambulance. • D: applying ice packs to her forehead. - ANS You selected B; this is correct! Reason: Patients with migraine or cluster headaches typically have photophobia (light sensitivity). Any type of bright light, especially if shone directly into the eyes, will cause the patient with a headache unnecessary severe pain. Dimming the lights in the ambulance and making the patient as comfortable as possible are the treatments of choice for a patient with a headache. Some patients benefit from ice packs applied to the forehead; just be sure to wrap the ice pack with roller gauze. Oxygen also should be administered as needed. Typically, the patient will prefer to lie supine or on the side A patient who overdosed on heroin would be expected to present with: • A: tachycardia. • B: hyperpnea. • C: hypotension. • D: dilated pupils. - ANS The correct answer is C; Reason: Heroin is a Schedule I (illegal) narcotic that is typically injected. As with all narcotics, legal or illegal, overdose causes depression of the central nervous system (CNS), resulting in a decreased level of consciousness; bradycardia; hypotension; and slow, shallow (reduced tidal volume) breathing. Hyperpnoea (deep breathing) would not be present in a patient who overdosed on a narcotic. In a narcotic overdose, the pupils are typically constricted (mitosis). Barbiturates, such as phenobarbital, are also CNS depressants and cause the same symptoms seen with narcotic overdose. The patient's pupils, however, are typically dilated (mydriasis), not constricted. When caring for any patient with a decreased level of consciousness, your primary concern should be the: • A: potential for airway compromise. • B: patient’s blood glucose level. • C: possibility of a spinal injury. • D: possibility of a drug overdose. - ANS You selected A; this is correct! Reason: Altered mental status could be caused by a high or low blood glucose level, drug overdose, or head injury, among other causes. Furthermore, the possibility of a spinal injury should be considered if the patient was injured. However, your primary concern should be the status of the patient's airway. Patients with a decreased level of consciousness are at risk for aspiration if vomiting occurs. Unless spinal trauma is present or the patient is breathing inadequately (egg, fast or slow rate, shallow breathing [reduced tidal volume]), place him or her in the recovery position to facilitate drainage if vomiting occurs. Remember this: no airway, no patient! A 16-year-old, 125-pound male ingested a bottle of aspirin approximately 20 minutes ago. Medical control orders you to administer activated charcoal in a dose of 1 g/kg. How much activated charcoal should you administer? • A: 54 g • B: 60 g • C: 51 g • D: 57 g - ANS You selected D; this is correct! Reason: First, you must determine the patient's weight in kilograms (kg). Either of the following formulae can be used to convert pounds to kilograms: Formula 1: weight (in pounds) ÷ 2.2 = weight in kg. Formula 2: weight (in pounds) ÷ 2 - 10% = weight in kg. On the basis of the above formulae, a 125-pound patient weighs 57 kg. Using formula 1, the equation is as follows: 125 (weight in pounds) ÷ 2.2 = 56.81 (57 [rounded to the nearest tenth]). Using formula 2, the equation is as follows: 125 (weight in pounds) ÷ 2 = 62.5 (63 [rounded to the nearest tenth] - 6.3 (10% of 63) = 56.7 (57 [rounded to the nearest tenth]). Since the drug order is for 1 g/kg, you should administer 57 g of activated charcoal to your 125-pound patient. Which of the following is the MOST significant finding in a patient with a severe headache? • A: Pain in both legs • B: Chest discomfort • C: Unilateral weakness • D: Abdominal tenderness - ANS You selected C; this is correct! Reason: Unilateral weakness (weakness on one side of the body) is a clinically significant finding in a patient with a headache because it could indicate a stroke (ischemic or hemorrhagic). Abdominal, chest, and leg pain are not common complaints associated with a headache, although they should be noted and investigated if they are present. When caring for a patient with severe hypothermia who is in cardiac arrest, you should: • A: perform rescue breathing only. • B: perform BLS and transport. • C: avoid using the AED. • D: hyperventilate the patient. - ANS The correct answer is B; Reason: Patients with severe hypothermia (core body temperature < 86°F) who are in cardiac arrest should be managed with basic life support (chest compressions and ventilations), passive external rewarming (i.e., removal of wet clothing, applying warm blankets) and rapid transport to the hospital where they can be actively rewarmed. Because cold muscle is a poor conductor of electricity, defibrillation, if indicated, should be limited to 1 attempt until the patient's body temperature has been increased. Cardiac arrest patients with severe hypothermia generally do not respond to defibrillation. Hyperventilation should be avoided as this may increase intrathoracic pressure and impair blood flow back to the heart. • A: float a buoyant backboard under him, remove him from the pond, and begin rescue breathing. • B: move him to a supine position by rotating the entire upper half of his body as a single unit. • C: stabilize his head as you remove it from the water and open his airway by tilting his head back. • D: grab him by his clothing, remove him from the pond, and assess for breathing and a pulse. - ANS The correct answer is B; Reason: After safely reaching a drowning victim, you should first turn him to a supine position by rotating the entire upper half of the body as a single unit; protect the cervical spine as you do this because a spinal injury cannot be ruled out. Open the patient's airway, without manipulating the neck, and begin rescue breathing while still in the water. Float a buoyant backboard under the patient, secure him to it, and remove him from the water. After removing the victim from the water, assess for a pulse. If the victim is pulseless, begin CPR, dry him off, and apply the AED as soon as possible. Which of the following patients with diabetes is the BEST candidate for oral glucose? • A: A confused patient who has cool, clammy skin • B: A semiconscious patient with pale, clammy skin • C: An unresponsive patient who took too much insulin • D: A confused patient with suspected hyperglycemia - ANS You selected A; this is correct! Reason: Oral glucose is indicated for patients with diabetes who are in insulin shock (hypoglycemic crisis), or for any patient with symptomatic hypoglycemia (as documented by glucometer). The patient must be conscious and alert enough to be able to swallow the glucose, which comes in a tube of gel. If the patient is unresponsive or otherwise unable to swallow the glucose, you should provide rapid transport, providing the appropriate airway management en route, and consider an ALS intercept. A 40-year-old woman presents with widespread hives that she noticed about 45 minutes after taking penicillin. She is conscious and alert and denies difficulty breathing. Her breath sounds are clear to auscultation bilaterally, her vital signs are stable, and her oxygen saturation is 94%. She tells you she is allergic to wasps and has an epinephrine auto-injector. You should: • A: assist her in administering epinephrine via her auto-injector. • B: give high-flow oxygen and administer 100 mg of Benadryl. • C: give supplemental oxygen and transport her to the hospital. • D: advise her that she can probably drive herself to the hospital. - ANS You selected C; this is correct! Reason: The patient is experiencing an allergic reaction, but she is not in anaphylactic shock. Urticarial (hives) is common to all allergic reactions, regardless of severity; however, wheezing and hypotension, which she does not have, are specific to anaphylaxis. Although she carries an epinephrine auto-injector for her allergy to wasps, she was not stung by a wasp. Furthermore, the absence of wheezing and hypotension negates epinephrine administration. Give supplemental oxygen (in a concentration sufficient to maintain a SpO2 of greater than 94%) and transport her to the hospital. Because it can take up to an hour for signs of a severe allergic reaction to manifest, the patient should not drive herself. Diphenhydramine (Benadryl), an antihistamine, is an appropriate drug based on her presentation; however, it is not typically carried on a BLS unit. Furthermore, the correct dose is 25 to 50 mg. Immediately following a generalized motor seizure, most patients are: • A: hyperactive. • B: confused. • C: apneic. • D: awake and alert. - ANS You selected B; this is correct! Reason: After a generalized (grand mal) motor seizure, the patient typically will be confused, sleepy, or in some cases, combative. This is referred to as the postictal phase. The patient's level of consciousness typically improves within 30 minutes. In many cases, the patient's respirations will be fast (tachypnea) following a seizure; this is the body's attempt to eliminate excess carbon dioxide that accumulated in the blood during the seizure. When assessing a conscious patient who overdosed on a drug, you should FIRST determine: • A: if there is a history of prior overdose. • B: the patient's weight in kilograms. • C: the type of medication ingested. • D: when the medication was ingested. - ANS You selected C; this is correct! Reason: When assessing a patient who has overdosed on a drug, you should first determine what was ingested, which will provide you with immediate information about whether or not the substance is toxic. You should then find out when the medication was ingested. This information will provide medical control (or poison control) with the information needed to direct the most appropriate treatment. The patient's weight also should be estimated in kilograms in the event that an antidote is required. Information regarding prior drug overdoses would normally be obtained during the SAMPLE history. Which of the following actions should be carried out during the primary assessment of an unresponsive patient? • A: Assessing the skin • B: Obtaining a blood pressure • C: Palpating the cranium • D: Auscultating the lungs - ANS You selected A; this is correct! Reason: Components of the primary assessment for both responsive and unresponsive patients include assessing and managing the airway and assessing and managing circulation, which includes controlling any major bleeding; assessing the rate, regularity, and quality of the pulse; and assessing the color, condition, and temperature of the skin. When restraining a violent patient, you should make sure that: • A: the patient is restrained using maximal force. • B: at least two EMTs restrain the patient. • C: consent for restraint has been obtained from a family member. • D: someone talks to the patient during the process. - ANS You selected D; this is correct! Reason: If a violent patient needs to be restrained, you must ensure the presence of at least four people (one per extremity). One of the EMTs should continuously talk to the patient to explain what is happening, even if the patient is not listening. Restraint is a last resort used to protect the EMT as well as the patient. Consent is not needed from a family member prior to restraining the patient. Just enough force to effectively restrain the patient is all that is required to prevent causing unnecessary injury. Immediately following a generalized seizure involving tonic-clinic activity, most patients: • A: have a slow heart rate because of cardiac depression during the seizure. • B: are unresponsive but gradually regain consciousness. • C: experience prolonged apnea and require positive-pressure ventilations. • D: return to a normal level of consciousness but cannot recall the event. - ANS You selected B; this is correct! Reason: Most generalized (grand mal) seizures last 3 to 5 minutes and are followed by a lengthy (5 to 30 minutes) period called the postictal phase, in which the patient is unresponsive at first but gradually regains consciousness. During the seizure itself, the nervous system releases excess adrenaline (epinephrine); therefore, tachycardia, even after the seizure has stopped, is common. Apnea is common during a seizure; however, breathing usually resumes after the seizure has stopped. Many patients are tachypnea following a seizure; this is the body's attempt to eliminate excess carbon dioxide that accumulated in the blood during the seizure. Unlike grand mal seizures, petit mal seizures (also called absence seizures) can last for just a fraction of a minute, after which the patient fully recovers immediately with only a brief lapse of memory of the event. Which of the following organs are contained within the right upper quadrant of the abdomen? • A: Liver and gallbladder • B: Stomach and gallbladder • C: Liver and spleen • D: Liver and stomach - ANS You selected A; this is correct! Reason: The liver and gallbladder lie within the right upper quadrant of the abdomen. Most of the stomach is within the left upper quadrant, as is the entire spleen. Reason: When treating a patient with an altered mental status, you must first ensure a patent airway and determine if the patient is breathing adequately. If the patient is breathing adequately, administer supplemental oxygen and continue your assessment. If the patient is not breathing adequately (i.e., fast or slow rate, shallow breathing [reduced tidal volume]), assist his or her ventilations. It is important to ask a family member, if available, how the patient normally acts; this will help establish a baseline. Before administering oral glucose, you should assess the patient's blood glucose level to determine if hypoglycemia is the cause of his or her problem and then decide if the patient is able to swallow the glucose, if indicated. If the patient is unable to swallow, do not administer oral glucose, even if he or she is hypoglycemic. Request a paramedic ambulance if possible so the patient can receive intravenous glucose. A patient who overdosed on methamphetamine would be expected to have all of the following clinical signs, EXCEPT: • A: agitation. • B: hypertension. • C: dilated pupils. • D: bradycardia. - ANS You selected C; the correct answer is D; Reason: Methamphetamine, an upper, stimulates the central nervous system, causing it to release excessive amounts of adrenaline (epinephrine). Epinephrine increases heart rate and blood pressure; therefore, the patient would experience tachycardia and hypertension. Other signs of methamphetamine overdose, which also indicate an adrenaline surge, include pupillary dilation, agitation, and hyperthermia. Bradycardia would be expected in patients who have overdosed on drugs that suppress, not stimulate, the central nervous system (egg, narcotics, benzodiazepines, barbiturates). A 50-year-old woman with a history of epilepsy is actively seizing. Care for this patient should focus primarily on: • A: protecting her from injury and ensuring adequate ventilation. • B: administering high-flow oxygen and requesting an ALS ambulance. • C: frequently suctioning her airway and carefully restraining her. • D: placing a bite block in between her molars and giving her oxygen. - ANS You selected A; this is correct! Reason: Seizure deaths are most frequently the result of hypoxia. When a person is actively seizing, he or she is not breathing adequately. Your primary focus when treating a seizure patient is to protect him or her from injury and to ensure adequate ventilation and oxygenation. Many seizing patients require assisted ventilation. Suction the oropharynx only if the patient has secretions in his or her mouth. Do NOT insert anything into the mouth of a seizing patient; doing so may cause an airway obstruction or damage the soft tissues of the mouth, resulting in bleeding. Do not attempt to restrain an actively seizing patient; doing so may result in musculoskeletal injuries. Request an ALS ambulance per your local protocols. When you arrive at a residence for a man who is "not acting right," you enter the house and find him sitting on his couch. Which of the following findings would be MOST indicative of an altered mental status? • A: Slurred speech. • B: Tired appearance. • C: Odor of alcohol. • D: Eyes are closed. - ANS You selected A; this is correct! Reason: Often, an altered mental status can be difficult to assess, especially if you do not know how the patient normally acts. However, there are key findings that should increase your index of suspicion. An abnormal speech pattern, such as slurring or incoherent words, can be the result of a diabetic problem, alcohol intoxication, or drug ingestion. All of these can cause an altered mental status. The odor of alcohol suggests intoxication as a potential cause of his problem, but cannot be quantified. Just because the patient's eyes or closed or he has a tired appearance does not necessarily indicate that he has an altered mental status. Hypoxia-induced unresponsiveness during a submersion injury is usually the result of: • A: a cardiac dysrhythmia. • B: associated hypothermia. • C: laryngospasm. • D: water in the lungs. - ANS You selected C; this is correct! Reason: When a patient falls into the water or becomes panicked when in the water, he or she begins to swallow large amounts of water. Even a small amount of water near the larynx can cause a spasm, which closes off the airway. This results in hypoxia, loss of consciousness, and cardiac dysrhythmias. If the patient is not removed from the water at once and treated aggressively, hypoxia and acidosis will eventually result in cardiac arrest. Which of the following conditions would MOST likely cause flushed skin? • A: Low blood pressure • B: Blood loss • C: Hypothermia • D: Exposure to heat - ANS You selected D; this is correct! Reason: Whenever the body temperature rises (i.e., heat exposure, fever), the peripheral blood vessels dilate, which draws warm blood to the skin and gives it a flushed (red) appearance. Blood loss, shock, low blood pressure (hypotension), and hypothermia generally cause the skin to become pale; these conditions cause peripheral vasoconstriction, which shunts blood away from the skin. Which of the following mechanisms cause respiratory and circulatory collapse during anaphylactic shock? • A: Bronchodilation and vasodilation • B: Bronchoconstriction and vasoconstriction • C: Bronchoconstriction and vasodilation • D: Bronchodilation and vasoconstriction - ANS You selected C; this is correct! Reason: During anaphylaxis, histamines released from the immune system cause two negative effects that result in shock (hypo perfusion): vasodilation, which causes the blood pressure to fall and bronchoconstriction, which impairs breathing. A 50-year-old female complains of severe pain to the right lower quadrant of her abdomen. You should: • A: palpate the left upper quadrant of her abdomen first. • B: keep her supine with her legs fully extended. • C: quickly palpate that area first to assess for rigidity. • D: suspect that she has an acute problem with her liver. - ANS You selected A; this is correct! Reason: When assessing a patient with abdominal pain, determine where the pain is most severe (in this case, the right lower quadrant) and then palpate the quadrant furthest from that area first (in this case, the left upper quadrant). If you palpate the painful area first, the patient is less likely to allow you to assess the remainder of his or her abdomen; it also causes the patient unnecessary pain. Pain to the right lower quadrant suggests a problem with the appendix (egg, appendicitis), not the liver. Patients with abdominal pain commonly prefer to lie on their side with their legs drawn into their abdomen; this position often provides pain relief by taking pressure off of the abdominal muscles. If the patient prefers this position, do not discourage it. A 30-year-old man with a history of schizophrenia cut his wrists and is bleeding profusely. He is confused, combative, and has slurred speech. With the assistance of law enforcement personnel, you and your partner physically restrain him in order to provide care and transport. In this situation, a court of law would MOST likely: • A: conclude that you should have had a court order to restrain. • B: determine that the patient had decision-making capacity. • C: agree that you and your partner are guilty of assault and battery. • D: consider your actions in providing care to be appropriate. - ANS You selected D; this is correct! Reason: An adult with decision-making capacity (i.e., a mentally competent adult) has the legal right to refuse medical treatment, even if that treatment involves lifesaving care. In psychiatric cases, however, a court of law would likely consider your actions in providing lifesaving care to be appropriate, particularly if you have a reasonable belief that the patient would harm him- or herself or others without your intervention. In addition, a patient who is in any way impaired, whether by mental illness, medical condition, or intoxication, may not be considered competent to refuse treatment and transport. If you are unsure of a patient's decision-making capacity, err on the side of treatment and transport. Few would argue that it would be easier to defend why you treated a patient than to justify or defend why you abandoned a patient. • A: perform a rapid head to toe assessment and prepare for transport. • B: auscultate over his epigastrium to assess for bowel sounds. • C: vigorously palpate his abdomen to assess for a pulsating mass. • D: transport at once and be prepared to treat him for severe shock. - ANS The correct answer is D; Reason: Given the onset and nature of the patient's pain (egg, sudden onset, tearing sensation), you should suspect that he has an acute dissection of the abdominal aorta. Administer high-flow oxygen and transport him at once. Acute aortic dissection can quickly cause an aneurysm, which could rupture and cause profound shock. Therefore, you must carefully monitor him and be prepared to treat him accordingly. Unnecessary or vigorous palpation of his abdomen could cause his aorta to rupture and should be avoided. A rapid head-to-toe assessment is indicated for trauma patients with a significant mechanism of injury and unresponsive medical patients; this patient falls into neither of these categories. Auscultating bowel sounds in the field is time-consuming, will yield little information, and only delays transport. Which of the following statements regarding the function of insulin is correct? • A: It stimulates the liver to release glucose into the bloodstream. • B: It facilitates the uptake of glucose from the bloodstream into the cell. • C: It causes the pancreas to produce glucose based on the body's demand. • D: It promotes the entry of glucose from the cell into the bloodstream. - ANS You selected B; this is correct! Reason: Insulin is a hormone produced by the beta cells in the Islets of Langerhans of the pancreas. It promotes the uptake of glucose from the bloodstream into the cells where it is used in the production of energy. Glucagon, a hormone produced by the alpha cells in the pancreas, facilitates the conversion of glycogen to glucose (glycogenolysis) in the liver. The liver does not produce glucose; it produces glycogen, a complex sugar that the body cannot utilize until it has been converted to glucose, a simple sugar. A 72-year-old woman is found unresponsive in her poorly ventilated home. Her skin is flushed, hot, and dry, and her respirations are rapid and shallow. She is wearing a medical alert bracelet that states she is a diabetic and is allergic to sulfa drugs. You should be MOST suspicious for: • A: acute ischemic stroke. • B: a diabetic complication. • C: heat stroke. • D: anaphylactic shock. - ANS You selected C; this is correct! Reason: The patient's signs and symptoms are consistent with classic heat stroke. Unlike exceptional heat stroke, which usually affects otherwise healthy people who exert themselves in the heat for long periods of time, classic heat stroke commonly affects children and older adults, and typically occurs when the patient is in a hot, poorly ventilated space for a prolonged period of time. Significant underlying medical problems (egg, hypertension, diabetes, heart disease) increase the patient's risk for classic heat stroke. Both exceptional and classic heat stroke present with hot, flushed skin; however, patients with exceptional heat stroke may have moist skin, usually from residual perspiration, whereas patients with classic heat stroke typically have dry skin. A 36-year-old male, who is a known diabetic, presents with severe weakness, diaphoresis, and tachycardia. He is conscious, but confused. His blood pressure is 110/58 mm Hg, pulse is 120 beats/min and weak, and respirations are 24 breaths/min. The glucometer reads error after several attempts to assess his blood glucose level. In addition to high-flow oxygen, medical control will MOST likely order you to: • A: transport only and closely monitor him. • B: assist the patient in taking his insulin. • C: give him a salt-containing solution to drink. • D: give at least one tube of oral glucose. - ANS You selected D; this is correct! Reason: The patient's signs and symptoms indicate hypoglycemia. When you are in doubt as to a patient's blood glucose level, you should err on the side of caution and give sugar; this is what medical control will likely order you to do. The patient, although confused, is conscious and will likely be able to swallow. Insulin is not administered to patients in the field, even if hyperglycemia is documented; EMTs and paramedics are usually not familiar with all of the different types of insulin and their respective doses, and profound hypoglycemia, potentially resulting in death, can occur if too much insulin is given. After giving oral glucose, reassess the patient's mental status and vital signs. After moving a hypothermic patient to a warmer area, your primary focus should be to: • A: prevent further body heat loss. • B: provide rapid rewarming. • C: assess his or her body temperature. • D: give warm, humidified oxygen. - ANS You selected A; this is correct! Reason: After moving a hypothermic patient to a warmer area, your primary focus should be to prevent further body heat loss. Remove wet or damp clothing and cover the patient with blankets (passive rewarming). It is especially important to cover the patient's head; most heat is lost around the head and neck. You should not, however, try to actively rewarm the patient (i.e., placing heat on or into the body). Rewarming too quickly may induce a lethal cardiac dysrhythmia and cardiac arrest. For this reason, active rewarming should be done in the controlled setting of a hospital. In some areas, medical control may call for the administration of warm, humidified oxygen; follow your local protocols. Assessing a hypothermic patient's core body temperature requires a special hypothermia thermometer, which many EMS systems do not carry. Rapid transport of a patient who ingested a large dose of Tylenol is important because: • A: it only takes a small dose of Tylenol to cause cardiopulmonary arrest. • B: liver failure usually occurs within 6 hours following a Tylenol overdose. • C: an antidote may prevent liver failure if administered early enough. • D: activated charcoal cannot be given to patients who ingested Tylenol. - ANS You selected C; this is correct! Reason: Acetaminophen (APAP), the active ingredient in Tylenol, is a safe drug if taken as directed. However, ingestion of more than 140 mg/kg in an adult can cause liver failure and death. Symptoms of APAP overdose do not present acutely; it can take up to a week before signs of liver failure are apparent. The antidote for acetaminophen poisoning is acetyl cysteine (Acetate, Mucomyst), which is only given at the hospital. However, it must be given promptly if liver failure is to be avoided. Activated charcoal can be given to patients with APAP overdose; however, it is only effective if the drug is still in the stomach. After 1 to 2 hours following ingestion, activated charcoal would likely be ineffective. Which of the following structures is responsible for regulating body temperature? • A: Cerebrum • B: Medulla oblongata • C: Hypothalamus • D: Cerebellum - ANS You selected B; the correct answer is C; Reason: The hypothalamus, which is located within the brainstem, regulates body temperature by acting as the body's thermostat. During a heat-related emergency, the hypothalamus can "reset" the body's normal temperature to a much higher temperature in response to the environment and the body's inability to eliminate heat. A young male experienced a sync opal episode after working in the heat for several hours. He is conscious and alert; has cool, clammy skin; and complains of nausea and lightheadedness. You should: • A: provide rapid cooling. • B: give him cold water to drink. • C: advise him to go home and rest. • D: transport him on his side. - ANS The correct answer is D; Reason: The patient is experiencing heat exhaustion and should be transported to the hospital for evaluation, especially since he experienced a sync opal episode (fainting). Because he is nauseated, he should not be given anything to drink and should be placed on his side during transport to prevent aspiration if he vomits. Loosen any restrictive clothing that may trap heat. Rapid cooling is indicated for patients with heatstroke, the signs of which include an altered mental status and hot, flushed skin (dry or moist). Which of the following is a physiologic effect of epinephrine when used to treat anaphylactic shock? • A: As a vasodilator, it increases the blood pressure. • B: As an antihistamine, it blocks chemicals that cause the reaction. • C: As a vasoconstrictor, it lowers the blood pressure. • D: As a bronchodilator, it improves the patient's breathing. - ANS You selected D; this is correct! • D: offer oxygen and transport him in a position of comfort. - ANS You selected D; this is correct! Reason: Although the patient is likely experiencing the flu, there are other diseases, some of which are communicable, that can cause similar symptoms. The patient is requesting EMS transport; failure to comply constitutes abandonment. Although he is tachycardia, the remainder of his vital signs are stable; therefore, requesting an ALS ambulance to the scene to start an IV is not necessary. Simply transport him in a position of comfort and offer him supplemental oxygen. If the patient is infected with HIV or hepatitis, he may choose to voluntarily disclose that information. However, to inquire about infection with such diseases is unethical. In general, you should avoid rewarming a frostbitten body part in the field if: • A: arrival at the emergency department will be delayed. • B: the affected part could refreeze after rewarming. • C: you are unable to obtain water that is at least 120°F. • D: a paramedic is not present to administer analgesia. - ANS The correct answer is B; Reason: Frostbitten body parts should not be rewarmed if there is a chance that they could refreeze after you have rewarmed them. If an extremity thaws and then refreezes, the amount of tissue and cellular damage may be worse than the damage caused by the initial freezing. A delay in getting the patient to the emergency department warrants rewarming. If rewarmed, the extremity should be immersed in water that is 105° to 112°F. Analgesia would certainly be a comfort to the patient, although its absence does not negate rewarming a frostbitten body part in the field. When caring for a patient with an acute behavioral crisis, your primary concern should be: • A: gathering all of the patient's medications. • B: obtaining a complete past medical history. • C: ensuring you and your partner's safety. • D: providing safe transport to the hospital. - ANS You selected C; this is correct! Reason: When caring for a patient with a behavioral or emotional crisis, your primary concern should be for your own personal safety as well as your partner's safety. Your ultimate goal is to transport the patient to the hospital safely. Gather as much medical history information as possible, but keep in mind that many patients experiencing an emotional or behavioral crisis will not readily provide this information. It is important to remember that patients with emotional or behavioral crises may appear calm initially; however, there is always the potential for them to turn violent. Unresponsiveness, shallow breathing, and constricted pupils are indicative of what type of drug overdose? • A: Marijuana • B: Amphetamine • C: Narcotic • D: Barbiturate - ANS You selected C; this is correct! Reason: Signs of a narcotic (opiate) overdose from drugs such as heroin, morphine (Astromorph, Dura morph), meperidine (Demerol), or codeine include altered mental status; slow, shallow breathing; pupillary constriction (mitosis), hypotension; and bradycardia. Narcotics are central nervous system depressants that, when taken in excess, suppress the vital functions necessary for life, such as breathing, heart rate, and blood pressure. Barbiturates produce the same effects; however, the pupils are typically dilated (mydriasis), not constricted. Marijuana and amphetamine drugs are central nervous system stimulants and would thus cause the patient to become restless or even combative. Propoxyphene (Darvon) is categorized as what type of drug? • A: Narcotic • B: Amphetamine • C: Benzodiazepine • D: Barbiturate - ANS The correct answer is A; Reason: Propoxyphene (Darvon) is in the narcotic (opiate) class of drugs. Other narcotics include heroin, morphine, codeine, and meperidine (Demerol). Phenobarbital is an example of a barbiturate. Drugs such as Valium, Klonopin, and Xanax are benzodiazepines. Amphetamines include drugs such as Ritalin, Adderall, and Focalin. A 60-year-old man presents with a severe right-sided headache. He is conscious and alert. As you are assessing him, he tells you that the left side of his body feels weak. His blood pressure is 190/100 mm Hg, pulse is 88 beats/min, and respirations are 14 breaths/min. His past medical history includes diabetes, arthritis, and hypertension. You should be MOST concerned with: • A: his current blood pressure. • B: the location of his headache. • C: his blood glucose level. • D: the presence of hemiparesis. - ANS You selected D; this is correct! Reason: Headaches are a common medical complaint and have numerous causes, most of which are not life-threatening. Common causes of a headache include migraine and cluster headaches, sinusitis, and muscle tension, among others. In some patients, however, a headache can signal a serious problem. In this patient, you should be most concerned with his unilateral weakness (hemiparesis); this is not a common finding in patients with otherwise benign headaches and suggests a serious underlying problem (egg, intracranial bleeding, and brain tumor). The fact that his headache is to the right side of his head and his weakness is to the left side of his body suggests that the problem lies in the right side of the brain. It is doubtful that his blood glucose level (BGL) is abnormal; he is conscious and alert. However, it would not be unreasonable to assess his BGL. His blood pressure, which is significantly elevated, could be causing his headache or may be the result of other factors, such as noncompliance with his antihypertensive medication, anxiety, or pain. Which of the following would MOST likely cause a rapid drop in a patient's blood glucose level? • A: Mild exertion after eating a meal • B: Forgetting to take prescribed insulin • C: Taking too much prescribed insulin • D: Eating a meal after taking insulin - ANS you selected C; this is correct! Reason: Compared to diabetic coma (hyperglycemic ketoacidosis, hyperglycemic crisis), insulin shock (hypoglycemia, hypoglycemic crisis) has a rapid onset. It is commonly caused when a patient accidentally takes too much prescribed insulin. Insulin is a fast-acting drug that rapidly causes glucose to exit the bloodstream and enter the cell. Other common causes of hypoglycemia include taking a regular dose of insulin but not eating or taking insulin and exercising heavily. Eating a meal after taking insulin typically does not cause a significant change in the patient's blood glucose level. If a person fails to take his or her insulin, glucose will not be able to enter the cell and will accumulate in the bloodstream (hyperglycemia). A 44-year-old woman was bitten on the ankle by an unidentified snake while working in her garden. She is conscious and alert, has stable vital signs, and denies shortness of breath. Her only complaint is a burning sensation at the wound site. Your assessment reveals two small puncture wounds, redness, and swelling. You should: • A: conclude that envenomation likely did not occur, provide reassurance, and allow a friend to take her to the hospital. • B: give supplemental oxygen, splint her leg to decrease movement, and keep her leg below the level of her heart. • C: elevate her leg, cover the wound with a dry sterile dressing, and apply an ice pack to reduce pain and swelling. • D: administer high-flow oxygen, apply a constricting band proximal to the bite, and use ice to prevent venom spread. - ANS You selected B; this is correct! Reason: Given the fact that the snake was not identified, you should assume that it was poisonous. Furthermore, the presence of puncture wounds, burning, redness, and swelling are suggestive of envenomation. Therefore, you should provide emergency care and transport the patient to the hospital. Treatment for a snake bite involves keeping the patient calm, administering oxygen, splinting the affected extremity to decrease movement (helps slow the spread of venom), keeping the extremity below the level of the heart, and transporting the patient to the hospital. Do NOT apply ice to a snake bite; it may constrict the blood vessels and force venom further into the bloodstream. The use of a proximal constricting band is controversial; if one is used, it should be loose, not tight. En route to the hospital, monitor the patient's vital signs and mental status, and be alert for vomiting. A 34-year-old female complains of persistent fever, fatigue, and night sweats. During your assessment, you note that she has purple blotches on her arms and legs. She MOST likely has: be reversed with early, high-quality CPR. The energy associated with lightning is composed of direct current (DC) of up to 200,000 amperes and a potential of 100 million volts or more; however, the duration of a lightning strike is short. Therefore, skin burns are usually superficial; full-thickness (third-degree) burns are rare. Lightning causes massive contraction of all the body's muscles, potentially resulting in long bone and spinal fractures. Although this can clearly increase morbidity, it is not the most common cause of lightning-related deaths. You are transporting a 30-year-old man who is experiencing an emotional crisis. The patient does not speak when you ask him questions. How should you respond to his unwillingness to speak? • A: Remain silent until the patient speaks to you. • B: Do not speak to the patient, even if he begins to speak to you. • C: Tell the patient that you cannot help if he won't talk. • D: Continually encourage the patient to talk to you. - ANS You selected A; this is correct! Reason: When dealing with a psychiatric patient who is silent and unwilling to speak to you, do not fear the silence. The patient simply does not wish to speak. You should not press the issue, for doing so may upset the patient. You should remain calm until the patient speaks to you, and then respond accordingly. An elderly man is found unresponsive in his kitchen. The patient's wife tells you that her husband has diabetes and that he took his insulin, but did not eat anything. You should suspect: • A: diabetic coma. • B: hypoglycemia. • C: ketoacidosis. • D: hyperglycemia. - ANS The correct answer is B; Reason: Several factors point to a field impression of hypoglycemia. First, the patient is known to have diabetes and second, he took his insulin but did not eat. Because insulin promotes the uptake of glucose into the cells, if the patient does not replace this glucose by eating, the glucose level in the blood will fall to dangerously low levels. Hypoglycemia can also occur when a patient takes too much of his or her insulin. Hyperglycemia occurs when a patient does not take his or her insulin. In such cases, glucose accumulates in the bloodstream and is not able to enter the cell. In the absence of glucose, the cells begin to metabolize fat, which results in the production of ketoacidosis and leads to hyperglycemic ketoacidosis (diabetic coma, hyperglycemic crisis). You and your partner arrive at the home of a 60-year-old man with shortness of breath. As you enter the residence, you find the patient sitting in his recliner; he is in obvious respiratory distress. As you approach him, he becomes verbally abusive, stating that it took you too long to get to his home. A small handgun is sitting on a table next to his recliner. You should: • A: verbally distract the patient as your partner attempts to retrieve the gun. • B: slowly back your way out of the residence and call law enforcement. • C: turn around, rapidly exit the residence, and notify law enforcement. • D: explain the reason for your delay as your partner administers oxygen. - ANS You selected B; this is correct! Reason: There is an immediate threat to you and your partner's safety! The patient is verbally abusive and has a firearm within arm's reach. In this situation, you and your partner should slowly back away from the patient, exit the residence, and notify law enforcement. Do NOT take your eyes off of the patient; if you turn your back, you may take a bullet in the back! Backing away gives you the opportunity to take cover if you see the patient reach for his gun. Remember, your safety comes before all else. A 55-year-old woman with a history of diabetes is found unresponsive with rapid, shallow respirations. The patient's husband tells you that he does not know when his wife last took her insulin. Management of this patient should include: • A: subcutaneous injection of insulin and 100% oxygen. • B: oral glucose and oxygen via no rebreathing mask. • C: assisted ventilations and oral glucose. • D: assisted ventilations and rapid transport. - ANS You selected D; this is correct! Reason: Without knowing if and when the patient last took her insulin, it is difficult to determine if she is experiencing diabetic coma or insulin shock. Nonetheless, her rapid, shallow respirations—which are likely not producing adequate tidal volume—should be treated with ventilation assistance. Because she is unresponsive and obviously unable to swallow, oral glucose is contraindicated. Assess the patient's blood glucose level and pass this information along to the hospital; if possible, arrange for a paramedic intercept so she can receive intravenous dextrose. If the patient is experiencing diabetic coma, insulin is what she truly needs; however, insulin is rarely, if ever, administered in the prehospital setting—even by paramedics. After ensuring adequate oxygenation and ventilation, transport the patient without delay. When treating an unresponsive man who was struck by lightning, you should: • A: manually stabilize his head and open his airway. • B: apply full spinal precautions before moving him. • C: begin CPR at once if he is apneic and pulseless. • D: ensure that you and the patient are in a safe place. - ANS You selected D; this is correct! Reason: Contrary to popular belief, lightning can (and does) strike in the same place twice. After lightning strikes, the ground remains electrically charged for a period of time; this increases the chance of a second strike within a short period of time. You must first ensure that you and the patient are safe by moving to a sheltered area, preferably indoors. After you have ensured the safety of yourself and the patient, begin treatment as dictated by the patient's condition. Do not let the life you save, or attempt to save, TAKE your own! You are assessing a 40-year-old male with an apparent psychiatric crisis. According to the man’s wife, he has a history of depression and schizophrenia and takes Zoloft and Zyprexa. A police officer is present at the scene. The patient, who is obviously scared, tells you that he sees snakes everywhere. You should: • A: let him know that going to the hospital will keep him safe from the snakes. • B: let him know that it is important for him to be transported as soon as possible. • C: tell him that you do not see any snakes, but they are obviously scaring him. • D: advise him that you suspect he has not been compliant with his medications. - ANS You selected C; this is correct! Reason: When caring for any patient with a behavioral emergency, your primary responsibility is to yourself, to stay safe. The patient in this scenario, although scared, is not violent. In cases such as this, you should be prepared to spend extra time with the patient; it may take longer to assess, listen to, and prepare the patient for transport. Just because he is experiencing an acute crisis does not mean that he has not been taking his medications; to make that assumption is merely speculation on your part. You must be honest, reassuring, and nonjudgmental. Let the patient tell you how he or she is feeling in his or her own words, and acknowledge any auditory or visual hallucinations. Do not, however, play along with his or her hallucinations; this is cruel and only reinforces that what the patient is hearing or seeing is real. Which of the following structures is NOT part of the endocrine system? • A: Gallbladder • B: Pancreas • C: Thyroid • D: Pituitary - ANS You selected A; this is correct! Reason: The gallbladder, which concentrates and stores bile, is not an endocrine organ; it is a digestive organ. Endocrine organs produce hormones, which regulate other body organs and systems. The thyroid regulates metabolism; the pancreas produces insulin and glucagon; and the pituitary gland, which is located within the brain, is the "master" endocrine gland and regulates the function of all endocrine glands in the body. Assessment and treatment of a patient with severe abdominal pain should include: • A: giving the patient small sips of water. • B: placing the patient in a supine position. • C: administering supplemental oxygen. • D: vigorously palpating the abdomen. - ANS You selected C; this is correct! Reason: Patients with severe abdominal pain should be given supplemental oxygen (in a concentration sufficient to maintain a SpO2 of greater than 94%) and immediate transport. Most patients with abdominal pain prefer to lie on their side with their knees drawn up into their chest (fetal position), which takes pressure off of the abdominal musculature and often affords them some relief. Vigorous palpation of the abdomen should be avoided as this will only cause the patient more pain; instead, gently palpate each of the four abdominal quadrants, palpating the most painful area last. Because the prescribed epinephrine auto-injector. If so, you should contact medical control and obtain permission (if required by local protocol) to assist the patient with the auto- injector. Your patient is a 75-year-old female who, according to her son, tripped on a throw rug and fell. The patient is conscious and alert, is sitting on the couch, and has a hematoma to her forehead. When you ask her what happened, her son interjects by saying, "I already told you, she tripped and fell." As you further question her, you find that she is hesitant to answer your questions and keeps looking at her son. You should: • A: interview the patient and her son separately if possible. • B: tell the son that you suspect his mother has been abused. • C: notify law enforcement and have the son removed. • D: ask the patient if someone hurt her intentionally. - ANS You selected A; this is correct! Reason: There are several indicators that suggest this patient's injury was intentionally inflicted: the son's interjection, her hesitance to answer your questions, and the fact that she keeps looking at her son. If possible, interview the patient and her son separately; doing say may reveal inconsistencies regarding the event, which would further increase your index of suspicion for abuse. You must not, however, accuse her son of abuse or insinuate such. If you are wrong, you could be liable for slander. At present, the son is not verbally or physically abusive; however, if he becomes abusive, you should notify law enforcement at once. Remember, you are legally obligated to report any suspicions of abuse or neglect to the emergency department physician. You should make every effort to convince the patient to consent to treatment and transport. Which of the following signs or symptoms is MOST suggestive of a systemic reaction following ingestion of a poison? • A: Painful or difficult swallowing • B: Burns around the mouth • C: Tachycardia and hypotension • D: Nausea and vomiting - ANS You selected C; this is correct! Reason: Once an ingested poison gets into the system, it can affect multiple organ systems. Signs that this is occurring include tachycardia or bradycardia, hypotension or hypertension, weakness, restlessness, and an altered level of consciousness, among others. Local effects of an ingested poison include nausea and vomiting as the poison irritates the gastric lining, burns in and around the mouth, and painful or difficult swallowing (dysphagia). A patient with a core body temperature of 94°F would MOST likely present with: • A: joint and muscle stiffness. • B: shivering and pale skin. • C: decreased muscle activity. • D: an altered mental status. - ANS The correct answer is B; Reason: Mild hypothermia is defined as a core body temperature (CBT) between 93.2°F and 96.8°F (34°C and 36°C). Mildly hypothermic patients are usually alert and shivering in an attempt to generate heat through muscular activity. Their skin is typically pale due to the body's constriction of blood vessels at the skin to retain heat. In moderate hypothermia (CBT between 86°F and 93.2°F [30°C and 34°C]), shivering stops and muscular activity decreases. As the CBT falls further, all muscle activity stops. In severe hypothermia (CBT < 86°F [30°C]), vital functions decrease (egg, level of consciousness, BP, pulse, respirations) and the patient is at risk for life-threatening cardiac dysrhythmias. The muscles become rigid and the patient appears stiff. The patient may appear dead; although a pulse is present, it may not be palpable. An 80-year-old woman has pain in the right upper quadrant of her abdomen and a yellow tinge to her skin. You should suspect dysfunction of the: • A: gallbladder. • B: liver. • C: pancreas. • D: spleen. - ANS You selected B; this is correct! Reason: Pain in the right upper quadrant and skin with a yellow tinge to it (jaundice) indicates a problem with the liver. Jaundice is the result of an excess level of bilirubin in the blood. Bilirubin, a yellow-colored substance, is the waste product that remains in the bloodstream after the iron is removed from the hemoglobin. One of the functions of the liver is to filter out waste, such as bilirubin, from the blood. If the liver is not functioning properly, bilirubin accumulates in the bloodstream. Dysfunction of the pancreas would result in possible fluctuations in the levels of blood glucose. Gallbladder inflammation (cholecystitis) typically produces right upper quadrant pain and/or referred pain to the right shoulder that occurs shortly after eating. Dysfunction of the spleen would cause left upper quadrant pain and/or referred pain to the left shoulder. A man is experiencing a severe allergic reaction after being stung by a scorpion. He does not have his own epinephrine; however, his wife is allergic to bees and has a prescribed epinephrine auto-injector. You should: • A:request an ALS unit to respond to the scene to administer epinephrine. • B:provide rapid transport and consider an ALS rendezvous. • C:assist the patient with the wife's prescribed epinephrine. • D:assist the patient with one half the usual dose of the wife's epinephrine. - ANS You selected B; This is correct! Reason:If a patient does not have a prescribed epinephrine auto-injector and is experiencing a severe allergic reaction, you should administer supplemental oxygen, assist the patient's ventilations if needed, and transport without delay. Closely monitor the patient's airway and breathing status en route and coordinate an ALS rendezvous if possible. If you carry an epinephrine auto-injector on your ambulance and your protocols allow you to administer it, do so without delay. Otherwise, the ALS unit will be able to administer epinephrine via the intramuscular or intravenous route. Never assist a patient with a medication that is not prescribed to him or her specifically. You are transporting a 35-year-old male who has a history of alcoholism. He stopped drinking 4 days ago and is now disoriented, diaphoretic, and tachycardic. You should be MOST concerned that he: • A:may have a seizure. • B:is severely dehydrated. • C:will develop a high fever. • D:will begin hallucinating - ANS You selected A; This is correct! Reason:A patient in alcohol withdrawal may experience frightening hallucinations, or delirium tremens (DTs), a syndrome characterized by restlessness, fever, disorientation, sweating, agitation, and even seizures. About 1 to 7 days after a person stops drinking or when alcohol consumption levels are suddenly decreased, DTs may develop. Patients with DTs are often dehydrated due to sweating, fluid loss, insufficient fluid intake, or vomiting, and their hallucinations can be extremely frightening. However, the possibility of a seizure, clearly the most life-threatening complication associated with DTs, should concern you the most. While triaging patients at a mass-casualty incident, you encounter a responsive middle- aged female with a respiratory rate of 26 breaths/min. What should you do next? • A:Triage her as immediate (red tag) • B:Administer high-flow oxygen at once • C:Assess for bilateral radial pulses • D:Assess her ability to follow commands - ANS You selected C; This is correct! Reason: According to the Simple Triage And Rapid Treatment (START) method, if you encounter an adult patient with a respiratory rate that is less than 10 breaths/min or greater than 29 breaths/min, you should triage him or her as immediate (red tag) and move to the next patient. However, if the patient's respiratory rate is between 10 and 29 breaths/min, you should assess his or her hemodynamic status by checking for bilateral radial pulses. If the patient's radial pulses are absent, triage him or her as immediate and move to the next patient. If the patient's radial pulses are present, assess his or her ability to follow simple commands. If the patient is able to follow simple commands, triage him or her as delayed (yellow tag) and move to the next patient. If the patient is unable to follow simple commands, triage him or her as immediate and move to the next patient. Treatment does not occur during the initial triage process. The ultimate goal of any EMS quality improvement program is to: • A:recognize all EMTs who demonstrate consistency in providing competent patient care. • B:provide protocols to all EMTs and hold them accountable if protocols are not followed. • C:deliver a consistently high standard of care to all patients who are encountered. • D:ensure that all personnel receive an adequate number of continuing education hours. - ANS You selected C; This is correct! Failure of the EMT to obtain consent from a responsive patient before taking his or her blood pressure may constitute: • A:assault. • B:negligence. • C:abandonment. • D:battery. - ANS You selected D; This is correct! Reason:Battery is defined as unlawfully touching another person without his or her consent. Obtaining consent from every responsive patient prior to rendering care is of paramount importance. Assault is defined as instilling fear into another person, but does not involve actually touching him or her. Negligence occurs when the EMT fails to act as another prudent EMT would have acted in the same or similar situation. Abandonment occurs when the EMT terminates patient care without the patient's consent or transfers care of a patient to a provider of lesser training. The EMT should avoid focusing all of his or her attention on a single critical patient during the triage process because: • A:the patient will most likely die before he or she can be transported. • B:all of his or her supplies will likely be depleted on that one patient. • C:three EMTs are required to effectively manage a critical patient. • D:other patients may die of problems that may have been corrected. - ANS You selected D; This is correct! Reason: Triage is the process of rapidly assessing patients in order to determine their treatment priority. Focusing your attention on one patient during the triage process not only defeats the purpose of triage (to do the greatest good for the greatest number of people), but it neglects other patients who may die from injuries or conditions that may have been corrected if detected earlier. You are cleaning the back of the ambulance after transporting a patient with major trauma. Which of the following contaminated items should NOT be placed in a plastic biohazard bag? • A:Suction canister • B:Plastic IV catheter • C:Rigid suction catheter • D:Blood-soaked gauze pads - ANS You selected B; This is correct! Reason:Contrary to popular belief, the end of a Teflon IV catheter (the plastic catheter without the needle) can cut you; it is sharp! Therefore, it should be placed in a puncture- proof sharps container, not a plastic biohazard bag. Items that are blood-soaked or are otherwise contaminated, but cannot puncture, can safely be placed in a plastic biohazard bag. At the scene of a mass-casualty incident, you notice a bystander who is emotionally upset. An appropriate action to take would be to: • A:tell the bystander to leave the scene at once. • B:assign the bystander a simple, non-patient-care task. • C:have the bystander assist you with patient care. • D:notify the police and have the bystander removed. - ANS You selected B; This is correct! Reason:One of the most effective ways to reduce stress in a bystander at the scene of a mass-casualty incident is to assign the bystander a task that is not related to patient care. This may involve assisting other bystanders who are having difficulties as well or providing water to the rescuers. An obviously distressed bystander should not simply be sent away from the scene, but should be looked at as a patient as well. Clearly, if the bystander becomes aggressive or violent, law enforcement personnel should get involved. Which of the following statements regarding the high-efficiency particulate air (HEPA) respirator is correct? • A:A HEPA respirator is necessary only if the patient with suspected tuberculosis is coughing. • B:Long sideburns or a beard will prevent the proper fit of a HEPA respirator. • C:A HEPA respirator should be placed on any patient with tuberculosis. • D:A surgical mask provides better protection against tuberculosis than a HEPA respirator. - ANS You selected B; This is correct! Reason:If you are caring for a patient with known or suspected tuberculosis (TB), regardless of whether the patient is coughing, you should place a surgical mask (or high-flow oxygen, if indicated) on the patient and a high-efficiency particulate air (HEPA) respirator (N-95 or higher) on yourself. Unlike a surgical mask, the HEPA respirator is specifically designed to prevent exposure to the bacterium that causes TB. A surgical mask, however, will reduce the transmission of germs from the patient into the air. Do not place a HEPA respirator on the patient; it is unnecessary and uncomfortable. Use of a HEPA respirator should comply with OSHA standards, which state that facial hair, such as long sideburns or beards, will prevent a proper fit. You arrive at the scene of a young male who was stabbed when a burglar broke into his house. Law enforcement officers are present. The patient, who is unresponsive with several stab wounds to the chest, is lying in a narrow space between a couch and coffee table. You should: • A:move the coffee table, document what you did, and begin treatment. • B:treat the patient where he is, without moving the coffee table. • C:obtain permission from law enforcement before moving any furniture. • D:move the coffee table, begin patient care, and notify a police officer. - ANS You selected D; This is correct! Reason:After ensuring your own safety, your primary responsibility when functioning at a crime scene is to provide patient care. Optimally, you should help preserve the scene at the same time. In this situation, however, the patient requires immediate treatment; he is unresponsive and has life-threatening injuries. It would be difficult to provide effective care in his present position, and notifying law enforcement before you move any furniture would unnecessarily delay care. Therefore, you should move the coffee table out of the way and begin treatment. When possible, notify a law enforcement officer of what you moved, where you moved it to, and why you moved it. You should also document this on the patient care report, AFTER you have cared for the patient. Which of the following scene size-up findings is LEAST suggestive of an unsafe environment? • A:The sound of breaking glass as you approach a residence • B:Screaming and yelling coming from inside a residence • C:Liquid draining from a car that struck a telephone pole • D:A large man standing in his yard awaiting your arrival - ANS You selected D; This is correct! Reason: When approaching a residence, findings that would suggest an unsafe scene include, among other things, the sound of breaking glass, screaming and yelling, and an unusual silence. Liquid leaking from a wrecked automobile should be assumed to be gasoline and, therefore, dangerous. Although intimidating in appearance, there is no correlation between a person's physical size and his or her potential for violence. You and your partner have secured a trauma patient to a long backboard and are preparing to lift the backboard onto the stretcher. When doing so, you should: • A:recall that most of the patient's weight is at the foot end of the backboard. • B:ensure that the strongest EMT is positioned at the head of the backboard. • C:lift the backboard from the sides instead of from the ends. • D:be sure to lift the backboard with the powerful muscles of your back. - ANS You selected B; This is correct! Reason:Since more than half of the patient's weight is distributed to the head end of a backboard, you should always ensure that the strongest EMT is at that position. This will reduce the risk of injury to less strong personnel as well as the risk of dropping the patient. The backboard should be lifted from the ends, not the sides; you have less control over the board if it is lifted from the sides. When lifting any patient, you should use the powerful muscles of your thighs, not your back, to lift. Keep your back straight and in a locked-in position. Upon arriving at the scene of an overturned tanker truck, you see a clear liquid leaking from the rear of the tank. The driver is still in the vehicle and you can see that his face is covered with blood. You should: • A:request fire department assistance if they are not already en route. • B:ensure that the ambulance is downwind and downhill from the tanker. • C:put on gloves, a gown, and a mask and quickly remove the driver. • D:approach the rear of the tanker to identify the type of fluid leaking. - ANS You selected A; This is correct! have injuries incompatible with life, the woman with snoring respirations and severe burns has the best chance for survival. Therefore, she has the highest treatment priority. Which of the following practices will provide you with the highest degree of safety when responding to an emergency call? • A:Always requesting fire department assistance • B:Routinely using the lights and siren • C:Wearing your seatbelt and shoulder harness • D:Asking the police to escort you to the scene - ANS You selected C; This is correct! Reason:Consistent safety practices, such as wearing your seatbelt and shoulder harness, driving with due regard for others, and avoiding excessive speed, will afford you the highest degree of safety when responding to an emergency call. Unless used to guide you to an area you are unfamiliar with, escort vehicles should be avoided. Escort vehicles are especially dangerous at intersections; as the escort vehicle proceeds through the intersection, other motorists often do not expect a second emergency vehicle to follow. As a result, they may pull out into the intersection, thinking it is clear, and collide with your ambulance (wake effect collision). The use of lights and siren increases the danger factor, especially if used in conjunction with excessive speed. Lights and siren ask for, not demand, the courtesy of the right of way. Which of the following situations would necessitate treatment using implied consent? • A:A 17-year-old pregnant woman with an isolated extremity injury • B:A 65-year-old man who is confused and suspected of having a severe stroke • C:A 25-year-old man who is restless and has severe chest pain and diaphoresis • D:An 18-year-old man who is now fully alert after receiving oral glucose - ANS You selected B; This is correct! Reason:A patient may be treated under the law of implied consent, also called the emergency doctrine, any time he or she is unresponsive or otherwise lacks decision- making capacity (ie, confused, under the influence of drugs or alcohol). In cases such as these, the EMT should assume that the patient would consent to treatment and transport if he or she were able to make an informed decision. Examples of such patients include those who are intoxicated or who otherwise have an altered mental status (ie, stroke, hypoglycemia). Patients younger than 18 years of age may also be treated under the law of implied consent, unless the patient is female and is emancipated or pregnant You are called to a residence for a woman in cardiac arrest. Shortly after starting CPR, the patient's husband presents you with an unsigned document that states "do not attempt resuscitation." You should: • A:contact medical control prior to continuing any resuscitative efforts. • B:stop all resuscitative efforts in accordance with the document. • C:continue CPR until you have contacted medical control for guidance. • D:stop CPR until the document can be validated by a physician. - ANS The correct answer is C; Reason:Do not attempt resuscitate (DNAR) orders are particularly challenging for EMS providers. When presented with documentation, especially if it does not appear to be valid (in this case, an unsigned document), you should err on the side of patient care and continue resuscitative efforts until medical control orders you to stop. In which of the following situations is an emergency move of a patient from his or her wrecked vehicle clearly indicated? • A:The patient appears unresponsive and a high-power line is lying across the hood. • B:The patient has an altered mental status; diaphoresis; and rapid, shallow breathing. • C:Gas is leaking from the vehicle and there is a small fire in the engine compartment. • D:Your primary assessment reveals that the patient has signs and symptoms of shock. - ANS The correct answer is C; Reason:An emergency move is indicated if you or the patient's life is in immediate danger. Gas leaking from the vehicle and a fire in the engine compartment are clear indicators that you and the patient's lives are in imminent danger. An emergency move involves grabbing the patient by the clothing, protecting his or her spine as much as possible, and dragging him or her from the vehicle to a safe place. The rapid extrication technique, which involves manually stabilizing the patient's head, applying a cervical collar, and removing the patient from the vehicle onto a long backboard, is indicated if the patient's condition is unstable and/or he or she is in need of treatment that requires a supine position. Never approach or touch a vehicle that is in contact with a high-power line; have the power company cut the power to the line first and then remove the patient from the vehicle. Proper body mechanics when lifting and moving a patient include: • A:keeping the weight as close to you as possible. • B:twisting at the waist when moving around a corner. • C:using the muscles of your lower back to lift. • D:maintaining a slight curvature of your back. - ANS You selected A; This is correct! Reason:General guidelines for safe lifting and moving include keeping the weight as close to your body as possible; keeping your back in a straight, locked-in position; using the muscles of your thighs to lift; and avoiding twisting when moving a patient around a corner. Back injuries are the most common injury sustained by the EMT and can be easily avoided if proper lifting and moving techniques are observed. Your partner, a veteran EMT of 20 years, has been showing up late to work with increasing frequency over the last several shifts. When he arrives, he is in a bad mood and is clearly not interested in being at work. His behavior is MOST consistent with: • A:acute stress. • B:burnout. • C:drug use. • D:delirium. - ANS You selected B; This is correct! Reason:Your partner's behavior is consistent with burnout. Burnout is a condition of chronic fatigue, irritability, and frustration that results from mounting stress over time. Although burnout typically manifests after years of service in EMS, some EMTs begin to experience it in a very short period of time, especially if they work in EMS systems with a high call volume and low morale. Some people with burnout abuse drugs or alcohol; if you suspect this, you should report it to your supervisor immediately. The best way to prevent burnout is to recognize the signs of stress and take action to reduce it. An acute stress reaction occurs in response to a sudden, unexpected event; it is clear that your partner has had many stressful events over his career. Delirium is an acute change in cognitive ability; it commonly results from conditions such as hypoxia, hypoglycemia, and drug toxicity. When requesting medical direction for a patient who was involved in a major motor- vehicle accident, you should do all of the following, EXCEPT: • A:describe the severity of damage to the patient's vehicle. • B:use radio codes to describe the situation. • C:use proper medical terminology when speaking. • D:question an order if it seems to be inappropriate. You selected A; - ANS The correct answer is B; Reason:When giving a report to medical control or requesting medical direction, you should avoid the use of radio codes, such as "10-50." The physician may not be familiar with such codes. There is clearly less risk of confusion if you use plain English. At the same time, you should use proper medical terminology, especially when describing the patient's injuries. Information regarding the severity of damage to the patient's vehicle is critical information and should be relayed to the physician; this can help him or her appreciate the significance of the situation. Do not be afraid to question an order that is contrary to your training or protocols; the physician may have simply made an error. Repeating an order back to the physician, word for word, will minimize the risk of this occurring. While assisting a paramedic in starting an IV on a patient, you are inadvertently stuck by the contaminated needle while attempting to place it in the sharps container. You should: • A:notify your supervisor and request an HIV test. • B:hold the paramedic liable for the needle stick. • C:cease patient care immediately. • D:seek medical care as soon as possible. - ANS You selected D; This is correct! Reason:If you receive an exposure while providing patient care, you should notify your supervisor or designated infection control officer and seek medical care as soon as possible. However, this should be done after you have completed patient care. You will be tested for bloodborne pathogens such as HIV and hepatitis B, and based on the degree of exposure, the physician may suggest immediate treatment. The paramedic did not intentionally stick you with the needle; therefore, you cannot hold him or her AED pads, the man's wife tells you that she wants you to let him die in peace. You should: • A:comply with her request and cease all resuscitative efforts. • B:cease resuscitation only if the AED does not indicate a shock. • C:perform rescue breathing only and contact medical control. • D:continue performing CPR and ask her if he has a living will. - ANS You selected D; This is correct! Reason:When faced with a situation in which a family member does not wish for you to attempt resuscitation of a loved one, you should inquire about the presence of a living will or out-of-hospital do not attempt resuscitation (OOH-DNAR) order. If a valid living will or OOH-DNAR order is produced, it is generally acceptable to cease resuscitative efforts; consult medical control as needed. If a valid living will or OOH-DNAR order is not available, the most prudent action would be to continue CPR and contact medical control. Even in the absence of such documentation, medical control may advise you to cease resuscitation based on the wishes of the family and the patient's medical history. When in doubt, err on the side of the patient and attempt resuscitation. Few would argue that it is preferable to defend why resuscitation was attempted as opposed to why it was not. How does a unified incident command system differ from a single incident command system? • A:In a unified incident command system, a single incident commander is identified and will function as such, regardless of the type of incident • B:In a unified incident command system, plans are made in advance by all agencies that assume a shared responsibility for decision making • C:In a unified incident command system, one agency with several incident commanders has the majority of responsibility for incident management • D:In a unified incident command system, a single person is in charge of the entire incident, even if multiple agencies respond to the scene - ANS The correct answer is B; Reason:Regardless of the type of incident command system (ICS) used, a single incident commander (IC) must be in charge. In a unified incident command system, plans are drawn up in advance by all cooperating agencies that assume a shared responsibility for decision making. The response plan should designate the lead and support agencies for several types of mass-casualty incidents (MCIs). For example, the Haz-Mat team will take the lead in a chemical leak and the medical team will take the lead in a multi-vehicle car crash. Large MCIs often require a unified incident command system. A single incident command system is one in which one person is in charge, even if multiple agencies respond to the scene. It is generally used with incidents in which one agency has the majority of responsibility for incident management. Ideally, it is used for short-duration, limited incidents that require the services of a single agency. After arriving at a mass-casualty incident where other ambulances are already present, you should notify the dispatcher and then: • A:initiate care for the most critically injured patients. • B:obtain information from the fire service commander. • C:repeat the triage process. • D:report to the staging area. - ANS You selected D; This is correct! Reason:Once you arrive at the scene of a mass-casualty incident where an incident command system has already been established, you should report to the staging area, the area designated for all incoming ambulances and resources. The staging officer will know where help is needed the most and will be able to direct your actions accordingly. You are caring for a 6-year-old child with a swollen, painful deformity to the left forearm. As you communicate with the parents of this child, you should: • A:make sure that they remain aware of what you are doing. • B:tell them that the child will be transported to the hospital. • C:ask them repeatedly how the child was injured. • D:use appropriate medical terminology at all times. - ANS You selected A; This is correct! Reason:When caring for any patient, it is important to keep both the patient and family aware of what you are doing. You should avoid medical terminology whenever possible because most laypeople will not understand what you are saying. The plain English approach is much more effective. When caring for children specifically, you should inform the parents of the need for ambulance transportation and why; doing so will provide them with the information necessary to make an informed decision. Asking the parents repeatedly how the child was injured may be construed by some as implying that the child was abused. You arrive at the scene where a man fell approximately 40 feet and landed on his head. He is unresponsive, has agonal gasps, and a weak carotid pulse. Further assessment reveals an open head injury with exposed brain matter. Upon identifying this patient as an organ donor, you should: • A:recognize that the patient's injuries disqualify him as an organ donor. • B:manage the patient aggressively and provide rapid transport. • C:provide rapid transport only because the patient likely will not survive. • D:request authorization from medical control not to initiate care. - ANS The correct answer is B; Reason:It is unlikely that the patient will survive his injury; however, he is still breathing, albeit very poorly, and has a weak carotid pulse. Therefore, you should begin immediate and aggressive treatment and transport him promptly, just as you would for any other critically injured patient. It would clearly be inappropriate to request authorization to provide no care at all. If it is determined by a physician that he will not survive his injury, his organs can potentially be harvested and save several lives. According to the United States Department of Transportation (USDOT), minimum staffing for a basic life support ambulance includes: • A:at least two EMTs in the patient compartment. • B:a minimum of two EMTs in the ambulance. • C:an EMT who functions as the driver. • D:at least one EMT in the patient compartment. - ANS You selected D; This is correct! Reason:The United States Department of Transportation (USDOT) requires at least one EMT in the patient compartment of a basic life support (BLS) ambulance. Although the driver does not have to be an EMT, it is preferable. Regardless, the person operating the ambulance must be able to safely and effectively operate an emergency vehicle. Regulations regarding minimum staffing of an ambulance vary from state to state. When is it MOST appropriate to complete your patient care report for a critically ill or injured patient? • A:Promptly after the primary assessment • B:As soon as all patient care activities are completed • C:After the ambulance has been restocked at the station • D:Any time before you arrive at the hospital - ANS You selected B; This is correct! Reason:Patient care activities, especially when the patient's condition is critical, take priority over the completion of your patient care report (PCR). Once all patient care activities have been completed, you can complete the PCR. This is usually accomplished at the hospital or immediately upon returning to quarters. While functioning at a large-scale terrorist incident, it is important for the EMT to: • A:use triage and base patient care on available resources. • B:avoid placing any casualty in a "delayed" treatment status. • C:identify the person or persons responsible for the event. • D:begin immediate treatment of the most critically injured. - ANS You selected A; This is correct! Reason:During a terrorist incident, the basic foundations of triage and patient care remain the same; however, the treatment can and will vary. Terrorist incidents can produce a single casualty, hundreds of casualties, or thousands of casualties. When presented with wide-spread mass casualties, you must remember situational awareness. What you do in one situation may not be appropriate for another situation. In large-scale terrorist incidents, it is important to use triage and base patient care on available resources. When triaging casualties, use the same triage process that you would for any other mass-casualty incident. Remain focused on providing the greatest good for the greatest number of people, not the person or persons responsible for the incident. As you step out of the ambulance at the scene of a nighttime motor vehicle crash on the highway, your MOST immediate concern should be: • A:the presence of oncoming traffic. • B:whether the car will catch on fire. • C:rapid assessment of all injured patients. • D:placing safety flares by the ambulance. - ANS You selected A; This is correct! eight with a follow-through. A simple figure eight knot is seldom used. The half hitch is not a secure knot by itself, which is why it is used in conjunction with other knots. The clove hitch is used to attach a rope firmly to a round object, such as a tree or fencepost. Most crashes involving ambulances occur: • A:at intersections. • B:at stop lights. • C:at stop signs. • D:on the highway. - ANS You selected A; This is correct! Reason:Intersection crashes are the most common and usually the most serious type of collision in which ambulances are involved. When approaching an intersection, you should come to a complete stop, look in both directions for pedestrians and other motorists, and then proceed with caution. Remember, your lights and siren do NOT give you the right of way; they ask other motorists for the courtesy of the right of way. If you proceed through an intersection without stopping and strike another vehicle that had the right of way, you will be held liable. Whether or not your lights and siren were in use at the time of the incident is irrelevant. While you are inside a crashed vehicle assessing a patient who is entrapped, the rescue team should be: • A:actively extricating the patient using whichever extrication method they deem necessary. • B:preparing for a simple extrication process since you were obviously able to access the patient. • C:awaiting specific instructions from you as to how to proceed with the extrication process. • D:assessing exactly how the patient is trapped and determining the safest way to extricate. - ANS You selected D; This is correct! Reason:Your ability to access the patient, with or without difficulty, does not indicate the extent of entrapment or method of extrication necessary. The EMT's job is to assess and treat the patient; the rescue team's job is to determine the degree of entrapment and decide how to extricate. As you are assessing the patient, providing any care that you can within the confines of the vehicle, the rescue team should be assessing the vehicle and the degree of patient entrapment; this will enable them to determine the safest, most effective extrication approach. It is critical for the EMT in the vehicle and the rescue team outside the vehicle to constantly communicate. Once the patient has been assessed and provided any immediate life-saving care, and the rescue team has determined the best extrication approach, extrication can begin. You are the first ambulance to arrive at the scene of a motor-vehicle crash. As you approach the scene, you see three patients, two who have been ejected from their vehicles and the other who is still in his vehicle. You should: • A:notify the local trauma center so they can prepare for the patients. • B:call medical control and apprise him or her of the situation. • C:request additional ambulances to respond to the scene. • D:begin triaging and treating the most critically injured. - ANS You selected C; This is correct! Reason:One of the most important aspects of the scene size-up is to determine the need for additional resources. A single EMS unit and two EMTs cannot effectively manage three patients, especially if they are critically injured. Therefore, you must first request additional ambulances at the scene. After requesting additional resources, begin the processes of triage and treatment. Notify the local trauma center early, if possible, so they can prepare to take care of the patients; however, your first priority is to maximize patient care at the scene by requesting additional ambulances. When called to the scene of a structural fire to stand by in case any injuries occur at the scene, you should: • A:depart the scene and return to service after the fire has been completely extinguished. • B:enter the structure with the firefighters so you can provide immediate care to any victims. • C:park the ambulance close to the fire so you can rapidly access any patients. • D:ensure that your ambulance does not block or hinder other arriving fire apparatus. - ANS You selected D; This is correct! Reason:When standing by at the scene of a structural fire, you must ensure that the ambulance is parked at a safe distance; this will not only keep you safe from the fire itself, but also minimize your risk of injury if the structure collapses. You must also ensure that the ambulance does not block or hinder access to the fire by fire apparatus that arrives after you. You should receive instructions from the fire officer in charge regarding the appropriate location to park the ambulance. It is not the EMT's job, nor is it safe, to enter a structure fire; if victims are located by firefighters, they will be brought to you. Do not depart the scene and return to service until the fire officer in charge releases you; many dangers still exist after the fire has been extinguished. When calling your radio report to the receiving hospital, you should: • A:include the patient's name. • B:break your report into 60-second increments. • C:only give your report to a physician. • D:be brief, concise, and factual. - ANS You selected D; This is correct! Reason:A radio report should be brief, concise, and factual. It should include the patient's age and sex, his or her chief complaint, associated assessment findings, vital signs, treatment that you provided, and the patient's response to your treatment. Avoid speculative statements regarding the patient's condition; report only what you know to be fact. Longer radio reports should be broken into 30-second increments; after 30 seconds, pause and ensure the listener heard your previous traffic. The patient's name is not vital to your report; thus, there is generally no need to disclose it. Unless you are requesting medical direction, it is acceptable, and routine practice, to give your report to a registered nurse. Following a call in which a 6-week-old infant in cardiac arrest did not survive, your partner is exhibiting significant anxiety and irritability. How can you MOST effectively help him? • A:Report his behavior to the medical director. • B:Tell him that he needs psychiatric help. • C:Recommend at least 12 hours of sleep. • D:Allow him to voice his feelings to you. - ANS You selected D; This is correct! Reason:Your partner is clearly having difficulty coming to terms with this call's bad outcome. As his partner, you can be most effective during this time by simply listening and allowing him to voice his feelings. Bad feelings should never be kept bottled up. If your partner is still having difficulties, a formal critical incident stress debriefing (CISD) may be needed. Some EMS providers may require psychiatric or psychologic assistance; however, this is not a determination that you can make. At least 8 hours of sleep is healthy; however, in the face of a crisis, sleep does not make the problem go away. Medical control has ordered you to administer one tube of oral glucose to a hypoglycemic patient. Immediately after receiving this order, you should: • A:ask medical control to repeat the order word for word. • B:repeat the order back to medical control word for word. • C:document the order on the prehospital care report. • D:administer the medication and reassess the patient. - ANS You selected B; This is correct! Reason:Immediately after receiving an order from medical control, you should repeat the order back to medical control word for word. This will ensure that you heard correctly and understand the order to be carried out. If you receive an order that seems inappropriate, you should ask the physician to repeat the order back to you for clarification. Reassess the patient after administering the medication and document the time and patient's response (good or bad) on your patient care report. Your actions at the scene of a critically injured patient who was shot during a robbery should include: • A:caring for the patient while manipulating the scene minimally. • B:performing a primary assessment only. • C:providing care when the police authorize you to. • D:starting immediate care as you would with any other patient. - ANS The correct answer is A; Reason:After ensuring your own safety, your priority while functioning at a crime scene is to provide care to the patient. However, you should make a reasonable effort to avoid manipulating the scene in order to preserve potential evidence. If furniture or other your immunizations will reduce your risk of contracting certain diseases if you are exposed. The information that would be of LEAST pertinence when educating the public on injury prevention is: • A:how to provide rescue breathing. • B:teaching children to wear bicycle helmets. • C:the proper usage of child safety seats. • D:building a childproof fence around a pool. - ANS You selected A; This is correct! Reason:The goal of an injury prevention program is just that, prevention. If rescue breathing is needed in a situation, the injury has already occurred. As EMS providers, we are consequence managers. Additionally, we have a responsibility to educate the public on how to avoid injuries in the first place. While en route back to your station from the emergency department, you discover that you forgot to include vital patient information on the front of your patient care report (PCR). Having left a copy of your PCR with the emergency department staff, you should: • A:notify the emergency department staff and advise them of your error. • B:document the information on the front of your original PCR. • C:include the information on an addendum and furnish it to the hospital. • D:do nothing, since you already left a copy of the PCR at the hospital. - ANS You selected C; This is correct! Reason:Once you leave a copy of your patient care report (PCR) with the hospital staff, you should not add information to the front of the report. If you discover that you forgot to include vital patient information, you should document the information on a separate addendum and furnish it to the emergency department, which can be sent by fax. The copy of your PCR you leave at the hospital and the original PCR should contain the same information; adding information to the front of the PCR after leaving a copy at the hospital makes the copy and original two separate documents. You have completed your patient care report and left a copy at the hospital when you realize that you forgot to document a pertinent finding on the front of the report. You should: • A:take no action and report the event to your supervisor. • B:attach an addendum to the original run report. • C:complete a new run report and add the information. • D:write the information on the original run report. - ANS You selected B; This is correct! Reason:If you discover that you forgot to include pertinent information on your patient care report (PCR) after leaving a copy at the hospital, you should write the information on a separate addendum and attach it to the original PCR. A copy of the addendum also should be sent to the receiving facility. Once you leave a copy of your PCR at the hospital, you should not add anything to the original. Legally, this would result in two different records for the same patient. Many EMS systems use electronic PCR (E-PCR) software, in which case the PCR is electronically submitted to the receiving facility upon completion. As with the hand-written PCR, you should add an addendum to the E-PCR if you forget to include pertinent information on the original. While treating a patient in cardiac arrest, you turn the AED on and attach the pads to the patient. However, when the AED begins to analyze the patient's cardiac rhythm, it signals "low battery" and then shuts off. The patient subsequently died. Which of the following statements regarding this scenario is MOST correct? • A:You and your partner may be held liable for negligence. • B:The crew that preceded you may be held liable for negligence. • C:Most errors associated with the AED involve equipment failure. • D:The manufacturer of the AED will be held liable for negligence. - ANS You selected A; This is correct! Reason:The most common errors that occur with the AED are the result of operator error (not equipment failure), usually because no one made sure that the batteries were fully charged when checking the ambulance at the start of the shift. Because the patient died, you and your partner could be held liable for negligence. Remember, the entire ambulance must be checked by the oncoming shift to ensure that all equipment is functional and that all supplies are present. Even though the preceding crew is morally responsible for not replacing the batteries, the legal ramifications will rest on you and your partner's shoulders. When providing care to multiple patients at the scene of a mass-casualty incident, your goal should remain focused on: • A:transporting patients to the hospital. • B:keeping all bystanders at a safe distance. • C:initiating CPR for those in cardiac arrest. • D:immobilizing all patients at the scene. - ANS You selected A; This is correct! Reason:At the scene a mass-casualty incident, you will be faced with many challenges, including ensuring your safety, extrication, triage, and patient care. In the midst of all of these activities, however, you must never lose sight of your ultimate goal, which is to transport all patients to the hospital as soon as possible. You arrive at the scene of an 80-year-old woman who is weak and lightheaded. Her son, who called 911, is present and asks you to transport his mother to the hospital. You should: • A:assess the woman and determine if she wishes to be treated and transported. • B:comply with the son's request and transport the woman to the hospital. • C:advise the son that he can probably drive his mother to the hospital. • D:take the woman's vital signs and apply supplemental oxygen if necessary. - ANS You selected A; This is correct! Reason:You must obtain consent from any mentally competent adult patient prior to initiating treatment. Just because the patient is 80 years old does not mean that she does not have decision-making capacity. And just because her son wants her to be transported does not mean that she does. Ask her if she wishes to be treated and transported to the hospital. If she does, then you have obtained consent and should proceed accordingly. If she does not, you should determine if she has decision-making capacity; that is, whether or not she is mentally competent. If she is determined to have decision-making capacity, then you cannot legally treat or transport her. If she does not have decision-making capacity (eg, she is confused, under the influence of drugs or alcohol), then you may treat and transport under the law of implied consent. It is not the EMT's decision to determine, let alone recommend, that a patient be taken to the hospital via privately owned vehicle (POV). If the patient requests EMS treatment and transport, you are legally obligated to do so. Which of the following actions demonstrates an EMT's knowledge of crime scene preservation? • A:Carefully cutting through the hole in a patient's clothing that was made by a large caliber firearm • B:Requesting approval from law enforcement before controlling severe bleeding from a patient's arm • C:Advising a law enforcement officer after moving a coffee table to access a critically injured patient • D:Placing a knife in a plastic zip-lock bag and giving it to a law enforcement officer for safe-keeping - ANS You selected C; This is correct! Reason:After ensuring your own safety, your priority when caring for a patient at a crime scene is to do just that, care for the patient. If you need to move a piece of furniture to gain access to a critically injured patient, move the furniture, treat the patient, and then advise a law enforcement officer of what you moved and where you moved it to. In this way, you are providing immediate care to the patient, but are remaining aware that the location of any obstacles between you and the patient may serve as evidence. Clearly, you are not going to request approval from a law enforcement officer before treating a critically injured patient. Items that may have fingerprints on them, such as knives or guns, should be placed in a paper bag; condensation can accumulate in plastic bags, potentially destroying any evidence. When removing clothing from a gunshot victim, you should make an effort to cut around (not through) the hole in the clothing that was made by the bullet. The hole in a patient's clothing may contain gunshot residue and can provide valuable information regarding the type of weapon used and the distance between the assailant and the victim. While caring for an emotionally disturbed 30-year-old male, he suddenly becomes violent and needs to be physically restrained. During the restraint procedure, you should: • A:approach the patient from the front and converge on him quickly. • B:maintain communication with him and closely monitor his airway. • C:ensure that at least three people are available to safely restrain him. patient. Prepare for immediate transport after the rapid head-to-toe assessment has been performed and the appropriate spinal precautions have been taken. Which of the following is the MOST practical method of standard precautions when treating multiple patients during a mass-casualty incident? • A:Asking each patient you treat if he or she has a communicable disease • B:Placing clean gloves over soiled gloves in between patient contacts • C:Thoroughly washing your hands in between patient contacts • D:Changing your gloves in between contact with different patients - ANS You selected D; This is correct! Reason:Although the most effective means of preventing the spread of disease involves thorough hand-washing, this is not practical at the scene of a mass-casualty incident; you usually do not have the time or facilities to do this. In a situation where you are treating multiple patients, you should change your gloves in between patients; this will help prevent cross-contamination. The concept of standard precautions is based on the assumption that ALL bodily fluids are potentially infectious; thus, there is no need to ask a patient if he or she has a communicable disease. Furthermore, to make such an inquiry is unethical. Upon arriving at a scene in which a tanker truck overturned and is spilling an unknown liquid on the ground, you should: • A:quickly identify the material. • B:turn off your warning lights. • C:park upwind from the scene. • D:stay downhill from the scene. - ANS You selected C; This is correct! Reason:At the scene of a potential or actual hazardous materials incident, you should park the ambulance in an area that is both upwind and uphill from the incident. However, you must be prepared to quickly relocate if the wind direction changes. Staying uphill is important because many hazardous materials collect in low-lying areas, such as valleys. After ensuring that you are in a safe place, attempt to identify the chemical involved by reading the placard on the tanker (with binoculars) and referencing the placard number in the emergency response guidebook (ERG). Who has ultimate authority for all issues regarding patient care at the scene of a mass- casualty incident? • A:Incident commander • B:Most experienced EMT • C:EMS medical director • D:Treatment officer - ANS You selected C; This is correct! Reason:The incident commander (IC) is responsible for all logistical and operational aspects of a mass-casualty incident (MCI), such as designating section officers and working in conjunction with other agencies (eg, police, fire, EMS). For all issues regarding patient care, the EMS medical director has ultimate authority. Although the treatment officer is responsible for overseeing all emergency care provided at the scene, and EMTs working in the treatment area are providing direct patient care, these personnel are still functioning under the physician's license. During an MCI, the IC (or his or her designee) is typically in contact with the medical director, who is located at the base station hospital, via mobile phone or two-way radio. In some cases, the medical director may be physically present at the incident. Following an apparent terrorist attack, numerous patients present with shortness of breath and persistent coughing. A green haze is noted in the area in which the patients are located. Which of the following agents should you suspect they were exposed to? • A:Tabun (GA) • B:Phosgene oxime • C:V agent (VX) • D:Chlorine (CL) - ANS The correct answer is D; Reason:The patient's signs and symptoms are indicative of a pulmonary (choking) agent, specifically chlorine (CL). Chlorine (CL) was the first chemical agent ever used in warfare. It has a distinct odor of bleach and creates a green haze when released as a gas. Initially, it produces upper airway irritation and a choking sensation. Later signs and symptoms include shortness of breath, chest tightness, hoarseness and stridor as the result of upper airway swelling, and gasping or persistent coughing. Phosgene, not to be confused with phosgene oxime (a blistering [vesicant] agent), is also a pulmonary (choking) agent. Tabun (GA) and V agent (VX) are examples of chemical nerve agents. Nerve agents are among the most deadly chemicals developed. Designed to kill large numbers of people with small quantities, nerve agents can cause cardiac arrest within seconds to minutes of exposure. Upon arriving at the scene of a crash involving a large truck, you immediately note the presence of an orange placard on the side of the tank that the truck is pulling. This indicates that the vehicle is carrying a/an ______________ agent. • A:explosive • B:flammable • C:corrosive • D:radioactive - ANS The correct answer is A; Reason:The color of a warning placard indicates the general classification of agent being carried, while the United Nations (UN) number in the center of the placard indicates the exact agent being carried. For example, a red placard bearing the UN number 1203 indicates gasoline; red indicates the classification (flammable), and 1203 indicates the exact agent (gasoline). Orange placards indicate explosive or blasting agents, placards that are half yellow and half white indicate radioactive agents, and black placards indicate corrosive agents. Use your emergency response guidebook (and binoculars, if appropriate) to identify both the classification and exact agent involved. After assisting a patient with her epinephrine auto-injector, you should: • A:place the device in a puncture-proof container. • B:place the device in a red biohazard bag. • C:give it to the patient to have it refilled. • D:replace the cover and place it in the trash can. - ANS You selected A; This is correct! Reason:After any device is used that has the potential for causing an accidental needle stick or is otherwise contaminated, it should be placed in a puncture-proof container, which usually is red and has a biohazard logo on it. The cover of the auto-injector should never be replaced, nor should a needle be recapped. Epinephrine auto-injectors are not refillable. In most states, the EMT is required to report which of the following occurrences? • A:Animal bite • B:Motor vehicle crash • C:Injury to a minor • D:Drug overdose - ANS You selected A; This is correct! Reason:Although each state may have slightly differing reporting laws, most require the EMT to report cases such as child or elderly abuse, sexual assault, animal bites, and injury that occurs during the commission of a crime. Injury to a minor is typically not a reportable case unless abuse is suspected. Motor vehicle crashes and drug overdoses are not reportable cases either, unless they occur during the commission of a crime. A patient presents with severe bradycardia, hypersalivation, vomiting, and excessive tearing. Which of the following agents would MOST likely cause his signs and symptoms? • A:Phosgene • B:Chlorine • C:Soman • D:Anthrax - ANS The correct answer is C; Reason:Nerve agents (eg, V agent [VX], sarin [GB], soman [GD], tabun [GA]) are among the most deadly chemicals developed. Designed to kill large numbers of people with small quantities, nerve agents can cause cardiac arrest within seconds to minutes of exposure. Nerve agents, discovered while in search of a superior pesticide, are in a class of chemical called organophosphates, which are found in household bug sprays, agricultural pesticides, and some industrial chemicals. Organophosphates block an essential enzyme in the nervous system, which cause the body's organs to become overstimulated. The mnemonic "DUMBELS" can help you recall the signs and symptoms of nerve agent exposure; it stands for Diarrhea; Urination; Miosis (constricted pupils); Bradycardia; Emesis (vomiting), Lacrimation (excessive tearing); and Seizures, Salivation, and Sweating. You can also use the mnemonic "SLUDGEM," which stands for Salivation, Lacrimation, Urination, Defecation, GI distress, Emesis, and Miosis. from the ICS jeopardizes lives and increases the risk of losing control over the situation. When you arrive at the scene in which the ICS has been activated, you should report to the staging area, where you will be directed to the area in which you are needed. Report to the section officer of that area, receive your instructions, and carry them out. When you have completed your assignment, you must return to the section officer for further instructions. Depending on the situation, you may be sent to another section. The sections that you work in and the responsibilities that you are given may change during the incident. At no time should you attempt to function independently (freelancing); this defeats the purpose of the ICS and puts lives in jeopardy. While caring for an injured patient, you remove blood-soaked clothing in order to treat his injuries. You should dispose of the clothing by: • A:leaving it at the hospital. • B:leaving it at the scene. • C:placing it in a regular trash can. • D:placing it in a biohazard bag. - ANS You selected D; This is correct! Reason: The appropriate method for disposing of soiled clothing or any other "nonsharp" contaminated item is to place it in a red biohazard bag. The biohazard insignia as well as the red color alerts others that the items within the bag are contaminated. During the triage process, which of the following injuries or conditions would classify a patient as a high priority? • A:Partial-thickness burns with no respiratory difficulty • B:A large avulsion to the arm and an altered mental status • C:Pulselessness and apnea • D:Unilateral femur fracture and tachycardia - ANS The correct answer is B; Reason:During triage, patients with an altered mental status, who are in shock, or who have problems with airway, breathing, or circulation, are potentially salvageable and are given immediate priority. Patients who are pulseless and apneic have low priority in a mass-casualty situation. If you focus your efforts on cardiac arrest patients, who will most likely not survive anyway, patients who could have potentially been saved will die as well. Remember, the goal of triage is to provide the greatest good for the greatest number of patients. The primary clinical feature associated with exposure to a vesicant agent is: • A:vomiting blood. • B:muscle twitching. • C:tachycardia. • D:skin blistering. - ANS You selected D; This is correct! Reason:The primary route of exposure of blister agents, or vesicants, is the skin. If vesicants are left on the skin or clothing long enough, they produce vapors that can enter the respiratory tract. Vesicants cause burn-like blisters to form on the victim's skin as well as in the respiratory tract (if inhaled). Vesicant agents include sulfur mustard (H), Lewisite (L), and phosgene oxime (CX). The symbols H, L, and CX are military designations. Vesicants usually cause the most damage to damp or moist areas of the body, such as the armpits, groin, and respiratory tract. You arrive at the scene of a traffic accident in which multiple vehicles are involved. You see at least two patients who are lying on the road and are not moving. You should: • A:begin triaging the patients. • B:request additional ambulances. • C:begin immediate patient care. • D:notify medical control for advice. - ANS You selected B; This is correct! Reason:As soon as you determine that there are more patients than you and your partner can effectively manage, you should immediately request additional help. Waiting until you are overwhelmed with critically injured patients is not the time to call for help. When in doubt, it is best to call for help. You can always cancel any incoming ambulances if you later determine that they are not needed. After you have called for assistance, you should begin triaging and caring for the patients to the best of your ability. Proper guidelines for safe reaching include all of the following, EXCEPT: • A:keeping your back in a locked-in position. • B:avoiding twisting of your back. • C:reaching no more than 30" in front of your body. • D:avoiding hyperextension of your back. - ANS You selected C; This is correct! Reason:Safe reaching practices are critical to the prevention of a back injury. When reaching, you should keep your back in a locked-in position. You should avoid twisting or hyperextending your back, and should reach no more than 15 to 20 inches in front of your body. Which position is MOST appropriate for a mother in labor with a prolapsed umbilical cord? • A:Supine with legs elevated • B:Left lateral recumbent • C:Supine with hips elevated • D:Left side with legs elevated - ANS You selected C; This is correct! Reason:When the umbilical cord is prolapsed, the infant typically slides down the birth canal and rests on top of the cord, shutting off its own oxygen supply. Placing the mother supine with her hips elevated will cause the baby to slide back into the birth canal slightly, thereby relieving pressure off of the cord. It may be necessary to insert your gloved fingers into the mother's vagina and lift the baby's head off of the cord. Give the mother high-flow oxygen and transport without delay. A lateral recumbent (on the side) position is appropriate for pregnant women without a prolapsed cord and will help prevent the occurrence of supine hypotensive syndrome, a condition in which the pregnant uterus compresses the inferior vena cava and compromises cardiac output. Which of the following statements regarding two-rescuer child CPR is correct? • A:A compression to ventilation ratio of 15:2 should be delivered without pauses in compressions to deliver ventilations • B:The chest should be compressed with one hand and a compression to ventilation ratio of 30:2 should be delivered • C:Compress the chest with one or two hands to a depth that is equal to one third the diameter of the chest • D:The chest should not be allowed to fully recoil in between compressions as this may impair venous return - ANS You selected C; This is correct! Reason:When performing two-rescuer CPR on a child (1 year of age to the onset of puberty [12 to 14 years of age]), the chest should be compressed with one or two hands (depending on the size of the child), and a compression to ventilation ratio of 15:2 should be delivered. It is important to compress the chest to an adequate depth—one third the anterior-posterior diameter of the chest (about 1 1/2" in the child). The chest should be allowed to fully recoil in between compressions in order to maximize venous return to the heart. If an advanced airway device (ie, ET tube, multilumen airway, supraglottic airway) is not in place, two rescuers should deliver "cycles" of CPR; the compressor should pause briefly so the ventilator can deliver two breaths. A compression to ventilation ratio of 30:2 is used for one-rescuer child CPR. After an advanced airway device has been inserted, "cycles" of CPR should not be performed; compressions should be continuous at a rate of at least 100/min and ventilations should be delivered at a rate of 8 to 10 breaths/min (one breath every 6 to 8 seconds). Your assessment of a mother in labor reveals that a fetal limb is protruding from the vagina. Management of this situation should include: • A:positioning the mother with her hips elevated, administering high-flow oxygen, and providing transport. • B:applying gentle traction to the protruding limb to remove pressure of the fetus from the umbilical cord. • C:positioning the mother in a semi-Fowler's position, administering oxygen, and providing transport. • D:giving the mother 100% oxygen and attempting to manipulate the protruding limb so that delivery can occur. - ANS The correct answer is A; Reason:Limb presentations represent a dire emergency for the newborn and do not spontaneously deliver in the field. You should position the mother in a manner so that her hips are elevated in an attempt to slide the infant slightly back into the birth canal and remove pressure from the umbilical cord. Administer high-flow oxygen to the mother, cover the protruding limb with a sterile sheet (or any clean sheet, if a sterile sheet is not available), and transport immediately. Do NOT pull on the protruding limb as this may cause injury to the newborn. that factors such as cold temperature can affect CRT. Early in shock, the heart and respiratory rates increase in an attempt to compensate for decreases in oxygen; this occurs in both children and adults. When these compensatory mechanisms fail, the blood pressure falls, and the patient enters a state of decompensated shock. For this reason, you should not rely upon a patient's blood pressure to determine overall perfusion; the blood pressure may be maintained, despite inadequate perfusion. Which artery should you palpate when assessing for a pulse in an unresponsive 6- month-old patient? • A:Radial • B:Brachial • C:Femoral • D:Carotid - ANS You selected B; This is correct! Reason:You should assess the brachial pulse in infants younger than 1 year of age. The carotid or femoral pulse can be assessed in children older than 1 year of age. A carotid pulse is difficult to locate in infants because they have minimal space between their head and shoulders. You are caring for a 6-year-old child with a possible fractured arm and have reason to believe that the child was abused. How should you manage this situation? • A:Call the police and have the parents arrested. • B:Transport the child to the hospital regardless of the parents' wishes. • C:Advise the parents that the child needs to be transported. • D:Inform the parents of your suspicions. - ANS You selected C; This is correct! Reason:The responding EMT must handle cases of suspected child abuse with great care. You must never accuse the parents or caregiver of abuse. If you are wrong, you could be held liable for slander. Actions that would suggest such accusation includes summoning the police to have the parents arrested. Instead, you should advise the parents or caregiver that the child needs to be transported by ambulance, even if the injury is not life-threatening. The goal is to get the child to safety; however, this must be done legally (with parental consent). In most cases, you only need the consent of one parent to transport the child. Once at the hospital, you must apprise the physician of your suspicions. The preferred method for inserting an oropharyngeal airway in a small child is to: • A:insert the airway as you would in an adult, but use an airway that is one size smaller than you would normally use. • B:open the airway with the tongue-jaw lift maneuver and insert the airway until you meet slight resistance. • C:depress the tongue with a tongue blade and insert the airway with the downward curve facing the tongue. • D:insert the airway with the curvature towards the roof of the mouth and then rotate it 180 degrees. - ANS You selected C; This is correct! Reason:Keeping in mind that a child's tongue is proportionately large, the preferred method for inserting an oropharyngeal (oral) airway is to use a tongue blade to depress the tongue and slide the airway straight in, with the downward curve of the airway facing the tongue, until it rests just beyond the curvature of the tongue. If you use an oral airway that is too small, it will not reach the curvature of the tongue and propel it forward. If you use an oral airway that is too large, it may obstruct the airway. If you meet resistance when inserting an oral airway, you are likely using an airway that is too large. A 3-year-old female presents with respiratory distress. She is conscious, crying, and clinging to her mother. She has mild intercostal retractions and an oxygen saturation of 93%. The MOST effective way of delivering oxygen to her involves: • A:ventilations with a flow-restricted, oxygen-powered device. • B:gently restraining her and assisting her ventilations. • C:asking the mom to hold an oxygen mask near her face. • D:a nonrebreathing mask with the flow rate at 6 to 8 L/min. - ANS You selected C; This is correct! Reason:Do not assume that a child will simply allow you to administer oxygen to him or her as you would to an adult. The child in this scenario, who is in respiratory distress and is mildly hypoxemic (SpO2 of 93%), should receive supplemental oxygen; however, it should be given in a nonthreatening manner. Agitating a sick or injured child causes an increase in oxygen consumption and demand, which may cause the child's condition to deteriorate. In this scenario, ask the child's mother to hold an oxygen mask near the child's face (blow-by oxygen). Closely monitor her condition and be prepared to assist her ventilations with a bag-mask device if she deteriorates. An oxygen flow rate of 6 to 8 L/min is too low for a nonrebreathing mask; a flow-rate of 12 to 15 L/min should be used. Do NOT use a flow-restricted, oxygen-powered ventilation device (FROPVD) on any child; doing so may cause severe gastric distention and lung injury. Allow the child to assume a position of comfort and transport. The MOST important initial steps of assessing and managing a newborn include: • A:drying and warming the infant and obtaining an APGAR score. • B:suctioning the airway and obtaining a heart rate. • C:clearing the airway and keeping the infant warm. • D:keeping the infant warm and counting respirations. - ANS You selected C; This is correct! Reason:In the initial steps of assessing and managing the newborn, the most important aspects include clearing the airway of amniotic fluid and making sure that the baby stays warm. The APGAR score should not be relied on as the initial indicator for resuscitation because it is not performed until the child is 1 minute old. Clearly, this is too long to wait before assessment and intervention. After the airway has been cleared and the newborn has been warmed, the respirations, heart rate, color, and oxygen saturation (SpO2) should be assessed and managed accordingly. A 3-year-old child has a sudden onset of respiratory distress. The mother denies any recent illnesses or fever. You should suspect: • A:lower respiratory infection. • B:foreign body airway obstruction. • C:croup. • D:epiglottitis. - ANS You selected B; This is correct! Reason:You should suspect a foreign body airway obstruction in any child who presents with an acute onset of respiratory distress in the absence of fever. Croup, epiglottitis, and lower airway infections (ie, bronchiolitis, bronchitis) commonly present with a fever. If the child is experiencing a mild airway obstruction, in which he or she is moving adequate air, has a normal level of consciousness, and pink skin, do not attempt to relieve the airway obstruction; doing so may result in a severe airway obstruction. Offer oxygen and transport the child to the hospital without delay. If signs of a severe airway obstruction are present (ie, ineffective cough, decreased level of consciousness, cyanosis), you should perform abdominal thrusts until the object is expelled or the child becomes unresponsive. If the child becomes unresponsive, perform chest compressions. During transport of a woman in labor, the patient tells you that she feels the urge to push. You assess her and see the top of the baby's head bulging from the vagina. You should: • A:ask the mother to take short, quick breaths until you arrive at the hospital. • B:apply gentle pressure to the baby's head and notify the hospital immediately. • C:advise your partner to stop the ambulance and assist with the delivery. • D:allow the head to deliver and check for the location of the cord. - ANS You selected C; This is correct! Reason:If, during transport, the mother begins to deliver the infant, your first action should be to advise your partner to stop the ambulance and assist you with the delivery. Delivery of a baby should never be attempted in the back of a moving ambulance. During delivery, you should apply gentle pressure to the top of the baby's head (be careful of the fontanelles) in order to prevent an explosive delivery. After the head is delivered, you should quickly run your fingers around its neck to determine if the cord is wrapped around its neck (nuchal cord). If a nuchal cord is not present, suction the baby's mouth and nose and continue with the delivery. If a woman is having her first child, the first stage of labor: • A:is usually the longest and lasts an average of 16 hours. • B:generally does not allow time for you to transport. • C:is shorter than in women who have had other children. • D:is typically very short and only lasts about 2 hours. - ANS You selected A; This is correct! Reason:There are three stages of labor: dilation of the cervix, delivery of the baby, and delivery of the placenta. The first stage begins with the onset of contractions and ends acknowledge your presence is an abnormal sign and indicates significant hypoxia. This child must therefore be managed aggressively to prevent respiratory arrest and subsequent cardiac arrest. Immediately upon delivery of a newborn's head, you should: • A:suction the nose. • B:suction the mouth. • C:dry the face. • D:cover the eyes. - ANS You selected B; This is correct! Reason:As soon as the newborn's head has delivered, you should first suction the mouth, then the nose. As the infant is forced through the birth canal, the thoracic cavity is squeezed, which causes the infant to expel amniotic fluid from the lungs. If this fluid is not thoroughly suctioned, it can be aspirated, resulting in inadequate ventilation and hypoxia. Immediately before or after suctioning the infant's airway, you should check for the presence of a nuchal cord (umbilical cord wrapped around the neck). You arrive at a residence shortly after a 4-year-old boy experienced an apparent febrile seizure. The child is alert and crying. His skin is flushed, hot, and moist. His mother tells you that the seizure lasted about 2 minutes. You should: • A:begin rapid cooling measures at once. • B:allow the mother to take her child to the doctor. • C:give him acetaminophen or ibuprofen. • D:provide supportive care and transport. - ANS The correct answer is D; Reason:Febrile seizures are common in children between the ages of 6 months and 6 years; they occur when the child's body temperature suddenly rises or when an already febrile child experiences an acute fever spike. Treatment for a child who has experienced a febrile seizure involves providing supportive care (eg, monitoring ABCs, administering oxygen as tolerated) and transporting to the hospital. Any infant or child who experienced a seizure should be transported by EMS. Although febrile seizures are typically self-limiting and are rarely life-threatening, there are other causes of fever and seizures, such as meningitis. The EMT is generally not authorized to administer medications unless directed by local protocol or direct medical control. Active cooling (eg, cool or cold water baths) should be avoided; a sudden lowering of the child's temperature may cause him to shiver, which may cause a sudden fever spike and induce another seizure. Which of the following assessment parameters is a more reliable indicator of perfusion in infants than adults? • A:Level of orientation • B:Pulse quality • C:Blood pressure • D:Capillary refill - ANS You selected D; This is correct! Reason:Capillary refill time (CRT) is a reliable indicator of perfusion in children less than 6 years of age. When the capillary bed (eg, fingernail, forehead) is blanched, blood should return to the area in less than 2 seconds. Because peripheral perfusion decreases with age, CRT is a less reliable indicator of perfusion in older children and adults. Note that cold temperatures can affect CRT. Pulse quality is reliable in patients of any age; weak or absent peripheral pulses indicate poor perfusion in anyone. Blood pressure is the least reliable indicator of perfusion in patients of any age; it usually does not fall until the body's compensatory mechanisms have failed. Assessing an infant's level of orientation is not possible; infants do not know who they are, where they are, what happened, and what day it is. When assessing an infant's mental status, note his or her level of alertness and interactivity (eg, tracking with his or her eyes, crying versus quiet). Upon delivery of a baby's head, you see that the umbilical cord is wrapped around its neck. Initial treatment for this condition should include: • A:clamping and cutting the umbilical cord. • B:keeping the cord moist and providing rapid transport. • C:trying to remove the cord from around the neck. • D:gently pulling on the cord to facilitate removal. - ANS You selected C; This is correct! Reason:If you can see the umbilical cord wrapped around the newborn's neck (nuchal cord) when the head delivers, you should gently attempt to slide the cord from around the neck. If this is unsuccessful, you should clamp and cut the cord and continue the delivery. You must never pull on the umbilical cord. In cases where the umbilical cord is prolapsed (the cord presents before the baby), you should make an attempt to move the fetus off of the umbilical cord; this often involves inserting your gloved fingers into the vagina and gently lifting the baby's head off of the cord. Cover the exposed cord with sterile moist dressings, administer oxygen to the mother, and transport without delay. After clearing the airway of a newborn who is not in distress, it is MOST important for you to: • A:obtain an APGAR score. • B:apply blow-by oxygen. • C:keep the newborn warm. • D:clamp and cut the cord. - ANS You selected C; This is correct! Reason:After ensuring a patent airway (ie, suctioning and positioning), it is extremely important to keep the newborn warm. Newborns cannot maintain body temperature very well and hypothermia can develop very quickly. Blow-by oxygen should be given if the newborn is breathing adequately, but has cyanosis to the face, neck, or trunk (central cyanosis). The umbilical cord should not be clamped and cut until the cord has stopped pulsating and the newborn is breathing adequately. The Apgar score, which is performed at 1 and 5 minutes after birth (and every 5 minutes thereafter), is not used to determine the need for or extent of resuscitation; respiratory effort, heart rate, skin color, and oxygen saturation (SpO2) are used to determine this. After an advanced airway device has been inserted in a 6-month-old infant in cardiopulmonary arrest, you should deliver ventilations at a rate of: • A:6 to 8 breaths/min. • B:10 to 12 breaths/min. • C:8 to 10 breaths/min. • D:12 to 20 breaths/min. - ANS The correct answer is C; Reason:After an advanced airway device (eg, ET tube, multilumen airway, supraglottic airway) has been inserted during cardiac arrest, ventilate the patient at a rate of 8 to 10 breaths/min (one breath every 6 to 8 seconds). This ventilation rate applies to all age groups, except the newborn. Healthcare providers often deliver excessive ventilation, particularly when an advanced airway device is in place. Excessive ventilation (eg, hyperventilation) is detrimental because it causes an increase in intrathoracic pressure, which impedes blood flow back to the heart and decreases coronary perfusion. Hyperventilation also increases the risks of regurgitation and aspiration in the patient who does not have an advanced airway in place. A 9-year-old girl was struck by a car while she was crossing the street. Your assessment reveals a large contusion over the left upper quadrant of her abdomen and signs of shock. Which of the following organs has MOST likely been injured? • A:Kidney • B:Liver • C:Pancreas • D:Spleen - ANS You selected D; This is correct! Reason:Abdominal trauma commonly occurs in children as the result of motor vehicle versus pedestrian accidents. The contusions over the left upper quadrant and the signs of shock suggest significant injury to the spleen. The liver lies in the right upper quadrant, and the pancreas and kidneys lie in the retroperitoneal space. Although the exact injury cannot be determined in the field, you must treat the patient for shock and provide rapid transport. A 7-year-old child has an altered mental status, high fever, and a generalized rash. You perform your assessment and administer supplemental oxygen. En route to the hospital, you should be MOST alert for: • A:respiratory distress. • B:combativeness. • C:hypotension. • D:convulsions. - ANS The correct answer is D; Reason:High fever and an alerted mental status indicate sepsis (severe infection). A generalized rash should alert you to the possibility of meningitis—a condition caused by infection and inflammation of the meninges that protect the brain and spinal cord. Children with meningitis are at risk for seizures (convulsions), usually due to increased intracranial pressure (ICP) and/or high fever; therefore, you must continually monitor the child's condition en route to the hospital and be prepared to treat seizures if they occur. Reason: After the AED delivers a shock, you should immediately begin or resume CPR, starting with chest compressions. Perform CPR for 2 minutes and then reanalyze the child's cardiac rhythm. If the AED states that a shock is advised, defibrillate without delay and then continue CPR. If the AED states no shock advised, resume CPR and reassess in 2 minutes. Do not check for a pulse after defibrillation or if the AED gives a no shock message; this only causes an unecessary delay in performing chest compressions. Continue CPR and cardiac rhythm analysis and defibrillation (if indicated) every 2 minutes until ALS personnel arrive or the patient starts to move spontaneously. Prevention of cardiac arrest in infants and small children should focus primarily on: • A:providing immediate transport. • B:avoiding upsetting the child. • C:ensuring adequate ventilation. • D:keeping the child warm. - ANS You selected C; This is correct! Reason:The most common cause of cardiac arrest in infants and children is failure of the respiratory system. Their hearts generally are healthy, and they rarely go into ventricular fibrillation (V-Fib). The key to preventing cardiac arrest in the majority of infants and children is to ensure adequate ventilation and oxygenation. A 29-year-old woman, who is 38 weeks pregnant, presents with heavy vaginal bleeding, a blood pressure of 70/50 mm Hg, and a heart rate of 130 beats/min. She is pale and diaphoretic, and denies abdominal cramping or pain. Her signs and symptoms are MOST consistent with a/an: • A:placenta previa. • B:abruptio placenta. • C:ruptured ectopic pregnancy. • D:ruptured ovarian cyst. - ANS The correct answer is A; Reason:Of the conditions listed, placenta previa would be the least likely to present with abdominal pain, although some patients may have pain or cramping. Placenta previa is a condition in which the placenta develops over and covers some or all of the cervix. As the cervix dilates, the vasculature that attaches the placenta to the uterine wall tears, resulting in vaginal bleeding that is often severe enough to cause shock. By contrast, abruptio placenta is a condition in which the placenta prematurely separates from the uterine wall; it is characterized by tearing abdominal pain, heavy vaginal bleeding, and shock. Placenta previa and abruptio placenta occur during the later stages of pregnancy. A ruptured ovarian cyst typically causes lower abdominal pain, often unilateral. Ectopic pregnancy, a condition in which the egg implants and grows outside the uterus (usually in a fallopian tube), is a first trimester condition; it is typically discovered between 8 and 10 weeks of pregnancy. If the ectopic pregnancy ruptures, the patient often presents with a sudden stabbing pain in the lower abdomen and shock due to intraabdominal hemorrhage. Which of the following parameters is the LEAST reliable when assessing the perfusion status of a 2-year-old child? • A:Skin color and temperature • B:Presence of peripheral pulses • C:Systolic blood pressure • D:Capillary refill time - ANS You selected C; This is correct! Reason:You should never rely on the systolic blood pressure when assessing the perfusion status of anyone. More reliable parameters include assessing peripheral pulses, capillary refill time (most reliable in children younger than 6 years of age), and the condition and temperature of the skin. Remember that the body's compensatory mechanisms work to maintain the blood pressure, so when it falls, this corresponds to decompensated shock. You are responding to a call for a 2-year-old child who fell from a second-story window. With the mechanism of injury and the age of the patient in mind, you should suspect that the primary injury occurred to the child's: • A:head. • B:lower extremities. • C:abdomen. • D:chest. - ANS You selected A; This is correct! Reason:Because a child's head is proportionately larger than the rest of the body when compared to an adult, the head commonly is the primary site of injury. This is especially true in fall-related injuries, in which gravity causes the head to precede the rest of the body. Head injury is a leading cause of traumatic death in infants and small children. Which of the following is a sign of an altered mental status in a small child? • A:Consistent eye contact with the EMT. • B:Inattention to the EMT's presence. • C:Recognition of the parents. • D:Fear of the EMT's presence. - ANS You selected B; This is correct! Reason:Typically, a small child will fear the presence of a stranger in his or her environment and will maintain constant eye contact with the stranger; therefore, inattentiveness to your presence should alert you to the presence of an altered mental status. Which of the following statements regarding pediatric anatomy is correct? • A:Relative to the overall size of the airway, a child's epiglottis is smaller. • B:The child's trachea is more rigid and less prone to collapse. • C:Airway obstruction is common in children because of their large uvula. • D:The occiput is proportionately larger when compared to an adult. - ANS You selected D; This is correct! Reason:Compared to adults, infants and small children have a proportionately larger head, specifically the occiput (back of the head). Therefore, when positioning an infant or a child's airway, padding in between the shoulder blades is often needed to maintain neutral alignment of the head. Infants and children are at risk for an airway obstruction because their entire airway is smaller, not because their uvula is large. An infant or a child's trachea is less rigid than an adult's; therefore, it collapses more easily during respiratory distress. Relative to the overall size of an infant or a child's airway, the epiglottis is larger; it is also floppier. When you begin to assess a woman in labor, she states that her contractions are occurring every 4 to 5 minutes and lasting approximately 30 seconds each. Which of the following questions would be MOST appropriate to ask next? • A:Have you had regular prenatal care? • B:At how many weeks gestation are you? • C:Has your bag of waters broken yet? • D:How many other children do you have? - ANS The correct answer is B; Reason:When assessing a patient in labor, the first question you should ask is how far along in the pregnancy she is. If she is at less than 37 weeks gestation (37 to 42 weeks is term), you should prepare for possible resuscitation of the newborn if delivery occurs in the field. Other questions, such as asking if her amniotic sac (bag of waters) has ruptured and whether or not she has received prenatal care, also can help you anticipate and prepare for potential complications. You should also inquire as to how many times the patient has been pregnant, regardless of whether she carried the baby to term (gravida), and the number of times she has carried a baby beyond 28 weeks, regardless of whether it was born dead or alive (para). You and your partner are performing CPR on an infant with suspected sudden infant death syndrome (SIDS). An important aspect in dealing with such cases is: • A:remembering that most infants with SIDS can be successfully resuscitated. • B:focusing all of your attention on the infant, with little parental interaction. • C:carefully inspecting the environment in which the infant was found. • D:discouraging the presence of the parents during your resuscitation attempt. - ANS You selected C; This is correct! Reason:When managing an infant with suspected sudden infant death syndrome (SIDS), you will be faced with three tasks: assessment and management of the infant, communicating with and providing emotional support to the family, and assessing the scene. When assessing the scene, you should note the position in which the infant was found, any signs that suggest the infant was recently ill (eg, medications, humidifiers), and the general condition of the house (ie, clean or dirty). Because most infants die of SIDS during the night and are not discovered until the next morning, resuscitation is futile and is generally not indicated. In some cases, you may begin CPR as another member of your team discusses the situation with the parents. CPR is typically stopped after it is evident to the parents that the infant is dead and that resuscitation would be Your assessment of a 5-year-old child reveals that he is unresponsive with a respiratory rate of 8 breaths/min and a heart rate of 50 beats/min. Treatment for this child should include: • A:high-flow oxygen via nonrebreathing mask and rapid transport. • B:back slaps and chest thrusts while attempting artificial ventilations. • C:positive-pressure ventilation, chest compressions, and rapid transport. • D:assisted ventilation with a bag-mask device and rapid transport. - ANS The correct answer is C; Reason:A heart rate less than 60 beats/min in an infant or child—especially when accompanied by signs of poor perfusion and inadequate breathing—should be treated with positive-pressure ventilation, chest compressions, and rapid transport. Respirations of 8 breaths/min and a heart rate of 50 beats/min will not maintain adequate oxygenation and perfusion in a child. Back slaps and chest thrusts are indicated for a responsive infant with a severe foreign body upper airway obstruction. Which of the following techniques represents the MOST appropriate method of opening the airway of an infant with no suspected neck injury? • A:Perform the technique as you would for an older child or adult. • B:Lift up the chin and hyperextend the neck. • C:Tilt the head back without hyperextending the neck. • D:Gently lift the chin while maintaining slight flexion of the neck. - ANS You selected C; This is correct! Reason:Opening the airway in infants and small children involves keeping the head in a neutral or slightly extended position. Because the occipital region (back of the head) of the skull is proportionately larger in infants and small children when compared to an adult, hyperextension of the neck can result in a reverse flexion of the neck and subsequent airway blockage. Following delivery of a newborn, the 21-year-old mother is experiencing mild vaginal bleeding. You note that her heart rate has increased from 90 to 120 beats/min and she is diaphoretic. In addition to administering high-flow oxygen, treatment should include: • A:uterine massage and transport. • B:internal vaginal pads and treating for shock during transport. • C:treating for shock and uterine massage during transport. • D:placing her on her left side and transport. - ANS You selected C; This is correct! Reason:Blood loss of up to 500 mL within the first 24 hours after delivery is considered normal and usually is well tolerated by the mother. However, any bleeding, regardless of the severity, with accompanying signs of shock, must be treated accordingly. In this case, you should apply high-flow oxygen, treat the patient for shock by elevating her legs (if allowed by local protocol) and providing warmth, and provide rapid transport to the hospital while massaging the uterine fundus en route. Placing the mother on her left side is appropriate before she delivers and prevents supine hypotensive syndrome. Dressings should never be packed into the vagina; placing pads into the vagina increases the risk for maternal infection. Which of the following signs is MOST indicative of inadequate breathing in an infant? • A:Heart rate of 130 beats/min • B:Sunken fontanelles • C:Expiratory grunting • D:Abdominal breathing - ANS You selected C; This is correct! Reason:Expiratory grunting in an infant or a child with a respiratory problem is an ominous sign; it indicates impending respiratory arrest. Grunting represents the child's attempt to maintain oxygen reserve in the lungs. Sunken fontanelles, the soft spots on the infant's skull, indicate dehydration. Because infants have a protuberant abdomen and rely heavily on their diaphragm to breathe, their abdomen appears to move more than their chest during breathing; this is a normal finding and is why infants are often referred to as "belly breathers." An infant or a child with inadequate breathing may be tachycardic at first; however, as hypoxia becomes more severe, bradycardia often occurs. Bradycardia in an infant or a child with a respiratory problem indicates impending cardiopulmonary arrest. In contrast to the contractions associated with true labor, Braxton-Hicks contractions: • A:consistently become stronger and are not alleviated by changing position. • B:may be intensified by activity and are accompanied by a pink discharge. • C:do not increase in intensity and are alleviated by a change in position. • D:generally follow rupture of the amniotic sac and occur with regularity. - ANS You selected C; This is correct! Reason:During pregnancy, the mother may experience false labor, or Braxton-Hicks contractions, in which there are contractions but they do not represent true labor. Unlike true labor contractions, Braxton-Hicks contractions do not increase in intensity, are not regular, and are typically alleviated by activity or a change in position. The contractions associated with true labor, once they begin, consistently get stronger and closer together and are regular; a change in position does not relieve the contractions. True labor is also commonly followed by, or in some cases preceded by, a rupture of the amniotic sac (bag of waters) and a pink or red vaginal discharge that is generally accompanied by mucus (bloody show). A 5-year-old boy was struck by a car when he ran out into the street. When you arrive at the scene and approach the child, you see him lying supine approximately 15 feet from the car. Based on the child's age and mechanism of injury, which of the following should you suspect to be his PRIMARY injury? • A:Head injury • B:Upper thorax injury • C:Lower leg injury • D:Pelvic injury - ANS The correct answer is D; Reason:Children are smaller than adults; therefore, when they are injured by the same mechanism of injury as an adult, the location of their injuries may differ from those of an adult. For example, when an adult is struck by a vehicle, the primary injury typically occurs at or below the knees, depending on the height of the bumper at the time of impact. Because the child is shorter, initial impact typically occurs at or near the pelvis. Secondary injury occurs when child's chest collides with the vehicle's grille. Tertiary injury occurs when the child strikes the side of his or her head on the pavement after being propelled away from the vehicle. In some cases, the child is pulled underneath the vehicle and is dragged. Upon delivery of the baby's head, you note that the umbilical cord is wrapped around its neck. You should: • A:immediately clamp and cut the cord and continue the delivery. • B:give the mother high-flow oxygen and transport her on her side. • C:keep the cord warm and moist and transport without delay. • D:make one attempt to gently remove the cord from around its neck. - ANS You selected D; This is correct! Reason:If the umbilical cord is wrapped around the baby's neck (nuchal cord), the EMT should make one attempt to gently remove the cord from around its neck. If this is unsuccessful, clamp and cut the cord and continue with the delivery. A nuchal cord can cause fetal asphyxia and must be treated immediately upon discovery. You have just delivered a baby girl. Your assessment of the newborn reveals that she has a patent airway, is breathing adequately, and has a heart rate of 130 beats/min. Her face and trunk are pink, but her hands and feet are cyanotic. You have clamped and cut the umbilical cord, but the placenta has not yet delivered. You should: • A:reassess the newborn every 5 minutes and transport after the placenta delivers. • B:massage the lower part of the mother's uterus until the placenta delivers. • C:give the newborn high-flow oxygen via a nonrebreathing mask and transport. • D:keep the newborn warm, give oxygen to the mother if needed, and transport. - ANS You selected D; This is correct! Reason: The newborn is stable and does not require care beyond providing thermal management and monitoring. Oxygen is indicated for the newborn if it has central cyanosis (cyanosis to the face and trunk), and should be delivered via the blow-by technique with the flowmeter set at 5 L/min. Cyanosis to the periphery of the body (eg, hands and feet), which is called acrocyanosis, is a normal finding in the newborn. You should not wait at the scene for the placenta to deliver; it can take up to 45 minutes for this to occur. Begin transport, keep the newborn warm, and give oxygen to the mother if indicated. Massaging the uterine fundus (top part of the uterus) is indicated for women with postpartum vaginal bleeding (ie, BOTH the baby and placenta have delivered). The function of the uterus is to: • A:house the fetus as it grows for 40 weeks. • B:provide oxygen and other nutrients to the fetus. • A:cover the arm with a sterile towel and transport immediately. • B:gently push the protruding arm back into the vagina. • C:encourage the mother to push and give her high-flow oxygen. • D:insert your gloved fingers into the vagina and try to turn the baby. - ANS You selected A; This is correct! Reason: On rare occasions, the presenting part of the fetus is neither the head nor buttocks, but a single arm or leg. This is called a limb presentation. You cannot successfully deliver such a presentation in the field. These infants usually must be delivered surgically. If you encounter a limb presentation, transport immediately. Cover the protruding limb with a sterile towel. If the mother feels the urge to push, you should discourage this; pushing may put pressure on the fetus, potentially causing injury. Never try to push the limb back in, and never pull on it. Place the mother on her back, with head down and pelvis elevated. Because both mother and fetus are likely to by physically stressed in this situation, give the mother high-flow oxygen. In order to maintain neutral alignment of an 18-month-old child's airway, you should: • A:pad in between the shoulder blades. • B:hyperextend the head. • C:ensure that the head is slightly flexed. • D:place a rolled towel behind the head. - ANS You selected A; This is correct! Reason:Infants and small children have proportionately large heads, specifically the occiput (back of the head). Therefore, it is often necessary to place padding in between the scapulae (shoulder blades) in order to ensure neutral alignment of the head. Padding behind the head places the child's head in the sniffing position, which is used to facilitate intubation. If the infant or child's head is hyperextended, the large occiput may push the head forward, resulting in hyperflexion. Flexing the child's head will collapse the trachea, resulting in obstruction of the airway. Which of the following is an abnormal finding? • A:Respiratory rate of 26 breaths/min in a 2-year-old child • B:Systolic BP of 100 mm Hg in a 10-year-old child • C:Rapid, irregular breathing in a newly born infant • D:Heart rate of 80 beats/min in a 3-month-old infant - ANS The correct answer is D; Reason:An infant between 1 month and 1 year of age typically has a heart rate between 100 and 160 beats/min; a heart rate less than 100 beats/min in this age group is considered to be bradycardic. Newborn infants normally have irregular breathing that ranges between 40 and 60 breaths/min. The systolic BP for a child between 6 and 12 years of age typically ranges between 80 and 110 mm Hg. The respiratory rate in a child between 1 and 3 years of age typically ranges between 20 and 30 breaths/min. A 4-year-old girl fell from a third-story window and landed on her head. She is semiconscious with slow, irregular breathing and is bleeding from her mouth and nose. You should: • A:suction her oropharynx, open her airway with the jaw-thrust maneuver, insert an oropharyngeal airway, and assist her ventilations. • B:open her airway with the jaw-thrust maneuver while manually stabilizing her head, suction her oropharynx, and assist her ventilations. • C:open her airway by carefully tilting her head back, suction her oropharynx, and administer high-flow oxygen via nonrebreathing mask. • D:manually stabilize her head, open her airway with the jaw-thrust maneuver, insert a nasopharyngeal airway, and suction her oropharynx. - ANS The correct answer is B; Reason:In any semi- or unconscious patient with a head injury, you should manually stabilize the head and open the airway with the jaw-thrust maneuver. If there are any secretions in the mouth, suction the oropharynx. If possible, insert a simple airway adjunct. The patient in this scenario is semiconscious and likely has an intact gag reflex; therefore, you should not attempt to insert an oropharyngeal airway. Conversely, you should not insert a nasopharyngeal airway in patients with a head injury, especially if there is fluid or blood draining from the nose (a sign of a skull fracture). After ensuring a patent airway, you should turn your attention to the patient's breathing. Slow, irregular breathing will not provide adequate minute volume and should be treated with ventilatory assistance. You are dispatched to a residence for a child having a seizure. When you arrive at the scene, the 4-year-old child's grandfather tells you that he has had several full body seizures over the past 20 minutes, but never woke up in between the seizures. The child's skin is hot and flushed. This is MOST indicative of: • A:a febrile seizure. • B:an absence seizure. • C:status epilepticus. • D:a focal motor seizure. - ANS The correct answer is C; Reason:Status epilepticus is defined as a prolonged (greater than 20 minutes) seizure or multiple seizures without a return of consciousness in between seizures. A febrile seizure is caused by an abrupt rise in body temperature, usually due to a non-life- threatening infection (eg, middle ear infection). Most febrile seizures last less than 5 minutes, have resolved by the time EMS arrives at the scene, and are not followed by a postictal period. The child in this scenario, although febrile (hot, flushed skin), did not experience a seizure caused by fever alone; you should suspect other causes of fever and seizures, such as meningitis. An absence seizure, also called a petit mal seizure, is characterized by a blank stare and an absence of tonic-clonic motor activity. Like febrile seizures, absence seizures are usually of short duration and are not followed by a postictal period. A focal motor seizure is isolated to one part of the body, such as an extremity, but can progress to a generalized (full body) seizure. A sudden onset of respiratory distress in a 5-year-old child with no fever is MOST likely the result of: • A:a foreign body airway obstruction. • B:inflammation of the upper airway. • C:infection of the lower airways. • D:a progressive upper airway infection. - ANS You selected A; This is correct! Reason:Children with no fever who have a sudden onset of respiratory distress should be treated for a foreign body airway obstruction. If the child is able to cough, cry, or speak, he or she is moving adequate air and has a mild airway obstruction. If the child is unable to cough, cry, or speak, and if he or she is cyanotic and has a decreased level of consciousness, he or she is not moving adequate air and has a severe airway obstruction. Epiglottitis, a bacterial infection of the upper airway, also causes a sudden onset of respiratory distress, but is accompanied by a high fever. Croup, a viral infection of the upper airway, typically does not present acutely; however, it is often accompanied by a low-grade fever. Lower airway infections (ie, bronchitis, bronchiolitis) typically present with a progessive onset of respiratory distress and abnormal lung sounds, such as wheezing and rhonchi. You arrive at the scene shortly after a 3-year-old female experienced a seizure. The child, who is being held by her mother, is conscious and crying. The mother tells you that her daughter has been ill recently and has a temperature of 102.5°F. What is the MOST appropriate treatment for this child? • A:Oxygen via nonrebreathing mask, place the child in a tub of cold water to lower her body temperature, and transport. • B:Oxygen via the blow-by technique, transport, and request a paramedic intercept so an anticonvulsant drug can be given. • C:Oxygen via nonrebreathing mask, avoid any measures to lower the child's body temperature, and transport at once. • D:Oxygen via the blow-by technique, remove clothing to help reduce her fever, and transport with continuous monitoring. - ANS You selected D; This is correct! Reason:As evidenced by her recent illness and fever (102.5°F), this child has likely experienced a febrile seizure. Appropriate treatment for the child following a febrile seizure involves ensuring a patent airway, administering oxygen (the blow-by technique is generally better tolerated in children than a mask), removing the child's clothing to facilitate heat loss, and transporting to the hospital. Avoid cooling the child with water; doing so may cause the child to shiver—a mechanism that produces body heat—which may cause an abrupt rise in body temperature and another seizure. Since the child is no longer seizing, an anticonvulsant drug is not indicated. A prolapsed umbilical cord is dangerous because the: • A:cord might pull the placenta from the uterine wall during delivery. • B:baby's head may compress the cord, cutting off its supply of oxygen. • C:mother may die of hypoxia due to compromised placental blood flow. • D:cord may be wrapped around the baby's neck, causing strangulation - ANS You selected B; This is correct! Reason:A prolapsed umbilical cord, a condition in which a portion of the umbilical cord delivers before the baby, is a dangerous condition; the baby's head may compress the
Docsity logo



Copyright © 2024 Ladybird Srl - Via Leonardo da Vinci 16, 10126, Torino, Italy - VAT 10816460017 - All rights reserved